2012 AMC 10/12 A Discussion
Go back to the Math Jam ArchiveA discussion of problems from the AMC 10/12 A, which is administered February 7. We will cover the last 5 problems on each test, as well as requested earlier problems on the tests.
Copyright © 2024 AoPS Incorporated. This page is copyrighted material. You can view and print this page for your own use, but you cannot share the contents of this file with others.
Facilitator: Dave Patrick
DPatrick
2012-02-08 19:10:04
Welcome to the 2012 AMC 10A/12A Math Jam!
Welcome to the 2012 AMC 10A/12A Math Jam!
DPatrick
2012-02-08 19:10:13
I'm Dave Patrick, and I'll be leading our discussion tonight.
I'm Dave Patrick, and I'll be leading our discussion tonight.
DPatrick
2012-02-08 19:10:20
Before we get started I would like to take a moment to explain our virtual classroom procedures to those who have not previously participated in a Math Jam or one of our online classes.
Before we get started I would like to take a moment to explain our virtual classroom procedures to those who have not previously participated in a Math Jam or one of our online classes.
DPatrick
2012-02-08 19:10:29
The classroom is moderated, meaning that students can type into the classroom, but these comments will not go directly into the room. These comments go to the instructors, who may choose to share your comments with the room.
The classroom is moderated, meaning that students can type into the classroom, but these comments will not go directly into the room. These comments go to the instructors, who may choose to share your comments with the room.
DPatrick
2012-02-08 19:10:43
This helps keep the class organized and on track. This also means that only well-written comments will be dropped into the classroom, so please take time writing responses that are complete and easy to read.
This helps keep the class organized and on track. This also means that only well-written comments will be dropped into the classroom, so please take time writing responses that are complete and easy to read.
DPatrick
2012-02-08 19:10:53
There are a lot of students here! As I said, only (a fraction of the) well-written comments will be passed to the entire group. Please do not take it personally if your comments do not get posted, and please do not complain about it. I expect this Math Jam to be much larger than our typical class, so please be patient with me---there are quite a few of you here tonight!!
There are a lot of students here! As I said, only (a fraction of the) well-written comments will be passed to the entire group. Please do not take it personally if your comments do not get posted, and please do not complain about it. I expect this Math Jam to be much larger than our typical class, so please be patient with me---there are quite a few of you here tonight!!
DPatrick
2012-02-08 19:11:15
Also, we won't be going through the math quite as thoroughly as we do in our classes -- I can't teach all the material for every problem as we go. Another difference between tonight and our regular online classes is that it is very unlikely that we'll be able to answer every single question you ask. We usually do in our classes, but we have a large number of students tonight! So, please go ahead and ask questions, but also please understand if we aren't able to answer them all!
Also, we won't be going through the math quite as thoroughly as we do in our classes -- I can't teach all the material for every problem as we go. Another difference between tonight and our regular online classes is that it is very unlikely that we'll be able to answer every single question you ask. We usually do in our classes, but we have a large number of students tonight! So, please go ahead and ask questions, but also please understand if we aren't able to answer them all!
DPatrick
2012-02-08 19:11:30
We do have two assistants tonight who can help answer some of your questions: Ariana Levin (ArianaL) and Luyi Zhang (redcomet46).
We do have two assistants tonight who can help answer some of your questions: Ariana Levin (ArianaL) and Luyi Zhang (redcomet46).
DPatrick
2012-02-08 19:11:43
They can answer questions by whispering to you or by opening a window with you to chat 1-on-1. However, due to the large size of the session tonight, they may not be able to get to you right away (or at all). Repeating your question over and over is more likely to annoy us than to get it answered faster, so please, just ask your question once and be patient, and please understand that we may not be able to answer all the questions tonight.
They can answer questions by whispering to you or by opening a window with you to chat 1-on-1. However, due to the large size of the session tonight, they may not be able to get to you right away (or at all). Repeating your question over and over is more likely to annoy us than to get it answered faster, so please, just ask your question once and be patient, and please understand that we may not be able to answer all the questions tonight.
DPatrick
2012-02-08 19:12:12
Please also remember that the purpose of this Math Jam is to work through the solutions to AMC problems, and not to merely present the answers. "Working through the solutions" includes discussing problem-solving tactics. So please, when a question is posted, do not simply respond with the final answer. That's not why we're here. We're going to work through the problems step-by-step, and comments that skip key steps or jump ahead in the problem, without providing explanation or motivation, will likely be ignored.
Please also remember that the purpose of this Math Jam is to work through the solutions to AMC problems, and not to merely present the answers. "Working through the solutions" includes discussing problem-solving tactics. So please, when a question is posted, do not simply respond with the final answer. That's not why we're here. We're going to work through the problems step-by-step, and comments that skip key steps or jump ahead in the problem, without providing explanation or motivation, will likely be ignored.
DPatrick
2012-02-08 19:12:31
We will work the last 5 problems from the AMC 10A, then the last 4 problems from the AMC 12A. (There's an overlap: #24 on the 10A is also #21 on the 12A.) After that, time permitting, I will take requests for some other problems for discussion.
We will work the last 5 problems from the AMC 10A, then the last 4 problems from the AMC 12A. (There's an overlap: #24 on the 10A is also #21 on the 12A.) After that, time permitting, I will take requests for some other problems for discussion.
DPatrick
2012-02-08 19:12:44
Let's get started!
Let's get started!
DPatrick
2012-02-08 19:13:00
We'll start with #21 on the AMC 10A:
We'll start with #21 on the AMC 10A:
DPatrick
2012-02-08 19:13:13
You'll notice that I'll always "sticky" the current problem under discussion to the top of the classroom window. You can resize the top region by dragging the horizontal gray bar that separates the top region from the main part of the classroom.
You'll notice that I'll always "sticky" the current problem under discussion to the top of the classroom window. You can resize the top region by dragging the horizontal gray bar that separates the top region from the main part of the classroom.
hutyputy66
2012-02-08 19:13:35
Make a diagram!
Make a diagram!
Polynomial
2012-02-08 19:13:35
First Draw a Diagram
First Draw a Diagram
pieofdoom51413
2012-02-08 19:13:35
Diagram
Diagram
Binomial-theorem
2012-02-08 19:13:35
To solve this one, I drew a picture.
To solve this one, I drew a picture.
DPatrick
2012-02-08 19:13:45
You might have done it by computing the coordinates of all the points and applying some formula. That works but is unnecessarily complicated.
You might have done it by computing the coordinates of all the points and applying some formula. That works but is unnecessarily complicated.
DPatrick
2012-02-08 19:13:49
We can start by trying to sketch a picture.
We can start by trying to sketch a picture.
DPatrick
2012-02-08 19:14:02
(Presumably you drew a similar picture. Yours might look different than mine if you oriented the axes differently.)
(Presumably you drew a similar picture. Yours might look different than mine if you oriented the axes differently.)
andrewjjiang97
2012-02-08 19:14:19
its a rectangle
its a rectangle
ForeverAce
2012-02-08 19:14:19
Oh look! It's a RECTANGLE!
Oh look! It's a RECTANGLE!
negativebplusorminus
2012-02-08 19:14:19
It's a rectangle!
It's a rectangle!
Orange-2728
2012-02-08 19:14:19
that looks suspiciously like a rectangle to me
that looks suspiciously like a rectangle to me
mathawesome
2012-02-08 19:14:19
Blue Rectangle!
Blue Rectangle!
DPatrick
2012-02-08 19:14:25
It sure looks like a rectangle.
It sure looks like a rectangle.
DPatrick
2012-02-08 19:14:32
Is it really a rectangle? Do we know for sure?
Is it really a rectangle? Do we know for sure?
Klu2014
2012-02-08 19:14:45
Do you have to prove that it is a rectangle?
Do you have to prove that it is a rectangle?
DPatrick
2012-02-08 19:14:59
That's the nice(?) thing about the AMCs: you don't necessarily have to *prove* anything.
That's the nice(?) thing about the AMCs: you don't necessarily have to *prove* anything.
DPatrick
2012-02-08 19:15:07
You could guess it's a rectangle and go on from there.
You could guess it's a rectangle and go on from there.
DPatrick
2012-02-08 19:15:22
In fact, let's do that and see what we get, and we'll come back to the question of whether it's a rectangle or not.
In fact, let's do that and see what we get, and we'll come back to the question of whether it's a rectangle or not.
DPatrick
2012-02-08 19:15:35
Are any of the side lengths of EFGH easy to compute?
Are any of the side lengths of EFGH easy to compute?
mathlover3737
2012-02-08 19:15:58
Find the length of GF!
Find the length of GF!
eb8368
2012-02-08 19:15:58
GF
GF
anastasia84
2012-02-08 19:15:58
GF
GF
Xu12345
2012-02-08 19:15:58
GF
GF
DPatrick
2012-02-08 19:16:08
GF is pretty easy to compute. How?
GF is pretty easy to compute. How?
iwantcombo
2012-02-08 19:16:37
they are just halves of the original side lengths
they are just halves of the original side lengths
mentalgenius
2012-02-08 19:16:38
yeah, GF = CB/2, and we find CB with pythag
yeah, GF = CB/2, and we find CB with pythag
bookie331
2012-02-08 19:16:38
GF is half of CB
GF is half of CB
DPatrick
2012-02-08 19:16:46
Notice that AFG is similar to ABC and half the size.
Notice that AFG is similar to ABC and half the size.
Iggy Iguana
2012-02-08 19:17:26
same thing with EF: 2 * EF = AD
same thing with EF: 2 * EF = AD
Watermelon876
2012-02-08 19:17:27
Similarly FE is half of AD
Similarly FE is half of AD
pieofdoom51413
2012-02-08 19:17:27
we can compute GH as well
we can compute GH as well
DPatrick
2012-02-08 19:17:58
Also CHG is similar to CDA and half the size. (Or BEF is similar to BAD and half the size.) Either way GH = FE = (1/2)(AD) = 3/2.
Also CHG is similar to CDA and half the size. (Or BEF is similar to BAD and half the size.) Either way GH = FE = (1/2)(AD) = 3/2.
DPatrick
2012-02-08 19:18:23
So let's go back to the earlier question: do we know that it's a rectangle? Or is it not?
So let's go back to the earlier question: do we know that it's a rectangle? Or is it not?
RelaxationUtopia
2012-02-08 19:19:05
Prove that the sides are perpendicular?
Prove that the sides are perpendicular?
joshy1227
2012-02-08 19:19:05
HG and EF are equal and parallel so yes
HG and EF are equal and parallel so yes
theone142857
2012-02-08 19:19:05
use the fact that FG and HE are both 1/2 and parallel to BC
use the fact that FG and HE are both 1/2 and parallel to BC
DPatrick
2012-02-08 19:19:23
Right, we just saw that EF and GH are parallel and the same length (they're both parallel to AD and half its length).
Right, we just saw that EF and GH are parallel and the same length (they're both parallel to AD and half its length).
DPatrick
2012-02-08 19:19:31
So it's at least a parallelogram.
So it's at least a parallelogram.
PestofWest
2012-02-08 19:20:06
Now we have to prove that the sides are perpendicular...
Now we have to prove that the sides are perpendicular...
jeff10
2012-02-08 19:20:07
prove one angle to be a right angle
prove one angle to be a right angle
2718euler
2012-02-08 19:20:07
GF lies in the XZ plane and so it is perpendicular to both GH and FE which lie in the YZ and XZ planes respectively.
GF lies in the XZ plane and so it is perpendicular to both GH and FE which lie in the YZ and XZ planes respectively.
DPatrick
2012-02-08 19:20:17
Furthermore, FG is on the xy-plane, and EF is parallel to the z-axis, so they're perpendicular.
Furthermore, FG is on the xy-plane, and EF is parallel to the z-axis, so they're perpendicular.
DPatrick
2012-02-08 19:20:26
Thus EFGH is a rectangle as we suspected, and our area computation was correct. The answer is (C).
Thus EFGH is a rectangle as we suspected, and our area computation was correct. The answer is (C).
DPatrick
2012-02-08 19:20:58
Let's move on to #22:
Let's move on to #22:
DPatrick
2012-02-08 19:21:29
We could just start plugging in formulas, but let's think for a moment first.
We could just start plugging in formulas, but let's think for a moment first.
Hydroxide
2012-02-08 19:21:38
the sum of the first m positive odd integers is m^2
the sum of the first m positive odd integers is m^2
NextEinstein
2012-02-08 19:21:38
sum of first m odd integers is m^2
sum of first m odd integers is m^2
andrewjjiang97
2012-02-08 19:21:38
1+3+5+...+2m-1=m^2
1+3+5+...+2m-1=m^2
ss5188
2012-02-08 19:21:38
sum of m integers is m squared
sum of m integers is m squared
DPatrick
2012-02-08 19:21:48
Good start: the sum of the first m positive odd integers is just m^2.
Good start: the sum of the first m positive odd integers is just m^2.
MCL
2012-02-08 19:22:12
sum of first n even integers is n(n+1)
sum of first n even integers is n(n+1)
scgorantla
2012-02-08 19:22:13
sum of first n positive even integers n(n+1)
sum of first n positive even integers n(n+1)
henrypickle
2012-02-08 19:22:13
sum of first n even integers is n(n+1)
sum of first n even integers is n(n+1)
Relativity1618
2012-02-08 19:22:13
sum of first n even integers is n(n+1)
sum of first n even integers is n(n+1)
DPatrick
2012-02-08 19:22:30
Right: another way to say "the sum of the first n positive even integers" is "twice the sum of the first n integers".
Right: another way to say "the sum of the first n positive even integers" is "twice the sum of the first n integers".
andrewjjiang97
2012-02-08 19:22:58
m^2=212+n(n+1)
m^2=212+n(n+1)
King6997
2012-02-08 19:22:59
m^2=212 + n(n+1)
m^2=212 + n(n+1)
DPatrick
2012-02-08 19:23:18
What do we do now?
What do we do now?
geoistrivial
2012-02-08 19:23:40
we form a quadratic with respect to n
we form a quadratic with respect to n
willwang123
2012-02-08 19:23:40
shove everything to one side and quadratic bash?
shove everything to one side and quadratic bash?
fermat007
2012-02-08 19:23:40
quadratic in terms of n
quadratic in terms of n
deepankar
2012-02-08 19:23:40
make a quadratic with n as the variable
make a quadratic with n as the variable
DPatrick
2012-02-08 19:23:47
I like this idea. Since we're trying to solve for n, we could write it as a quadratic in n:
I like this idea. Since we're trying to solve for n, we could write it as a quadratic in n:
flyrain
2012-02-08 19:24:13
use quadratic formula
use quadratic formula
electron
2012-02-08 19:24:13
and apply the quadratic formula?
and apply the quadratic formula?
Hydroxide
2012-02-08 19:24:15
The discriminant must be a perfect square.
The discriminant must be a perfect square.
Iggy Iguana
2012-02-08 19:25:04
we can say that 4m^2-847=a^2
we can say that 4m^2-847=a^2
theone142857
2012-02-08 19:25:04
set it to be b^2
set it to be b^2
DPatrick
2012-02-08 19:25:11
I like giving it a name too.
I like giving it a name too.
King6997
2012-02-08 19:25:30
difference of squares
difference of squares
2718euler
2012-02-08 19:25:31
Difference of Squares
Difference of Squares
Relativity1618
2012-02-08 19:25:31
difference of squares
difference of squares
DPatrick
2012-02-08 19:25:36
You said the magic words! :)
You said the magic words! :)
DPatrick
2012-02-08 19:25:57
(a **difference** of squares would be even better than what I wrote)
(a **difference** of squares would be even better than what I wrote)
fermat007
2012-02-08 19:26:12
847 = 7*11^2
847 = 7*11^2
va2010
2012-02-08 19:26:13
Factor 847
Factor 847
LightningStreak
2012-02-08 19:26:13
Find factors of 847!
Find factors of 847!
DPatrick
2012-02-08 19:26:23
Again, both m and a are integers! So we can look at factors of 847.
Again, both m and a are integers! So we can look at factors of 847.
DPatrick
2012-02-08 19:26:30
847 = 7 * 11 * 11.
So we have 847 = 1*847 = 7*121 = 77*11. (There are negative factorizations too, but they don't give us any different values of m^2.)
847 = 7 * 11 * 11.
So we have 847 = 1*847 = 7*121 = 77*11. (There are negative factorizations too, but they don't give us any different values of m^2.)
mistere
2012-02-08 19:27:02
Find the average of the factors
Find the average of the factors
Iggy Iguana
2012-02-08 19:27:14
to find m, sum the factor pairs and divide by 4
to find m, sum the factor pairs and divide by 4
DPatrick
2012-02-08 19:27:23
We could, but we really care about "a".
We could, but we really care about "a".
DPatrick
2012-02-08 19:27:41
We notice that 2a is the difference of the factors.
We notice that 2a is the difference of the factors.
yankeefan6795
2012-02-08 19:28:36
255
255
Iggy Iguana
2012-02-08 19:28:36
211+28+16=255
211+28+16=255
BookwormB
2012-02-08 19:28:36
then just add the possible values of n, and you're done :)
then just add the possible values of n, and you're done :)
ktbroborg
2012-02-08 19:28:36
211+28+16=255
211+28+16=255
DPatrick
2012-02-08 19:28:40
So our answer is 211 + 28 + 16 = 255, answer (A).
So our answer is 211 + 28 + 16 = 255, answer (A).
DPatrick
2012-02-08 19:29:09
A quick reminder since lots of people have been asking:
A quick reminder since lots of people have been asking:
DPatrick
2012-02-08 19:29:39
A complete transcript of the session will be up on the website after we're done. So if you have to leave, or parts are going too fast and you want to review later, look on the website in a couple of hours.
A complete transcript of the session will be up on the website after we're done. So if you have to leave, or parts are going too fast and you want to review later, look on the website in a couple of hours.
DPatrick
2012-02-08 19:29:55
Let's move on to #23 on the 10A, which is also #19 on the 12A:
Let's move on to #23 on the 10A, which is also #19 on the 12A:
DPatrick
2012-02-08 19:30:17
We need to first take a moment to make sure we understand the problem.
We need to first take a moment to make sure we understand the problem.
DPatrick
2012-02-08 19:30:29
There are 6 people (A,B,C,D,E,F so I don't have to write the names all the time). Each is friends with the same number of people.
There are 6 people (A,B,C,D,E,F so I don't have to write the names all the time). Each is friends with the same number of people.
joshy1227
2012-02-08 19:30:38
well they all dont have 5 internet friends, because then they all would be friends with eachother
well they all dont have 5 internet friends, because then they all would be friends with eachother
Iggy Iguana
2012-02-08 19:30:41
everyone can have 1, 2, 3, or 4 friends
everyone can have 1, 2, 3, or 4 friends
DPatrick
2012-02-08 19:30:56
Indeed, let's call the number of people that each is friends with n. We clearly have 0 <= n <= 5.
Indeed, let's call the number of people that each is friends with n. We clearly have 0 <= n <= 5.
DPatrick
2012-02-08 19:31:02
But the "Some, but not all" clause rules out n=0 and n=5. So we only have n=1, n=2, n=3, or n=4.
But the "Some, but not all" clause rules out n=0 and n=5. So we only have n=1, n=2, n=3, or n=4.
willwang123
2012-02-08 19:31:11
casework!
casework!
Klu2014
2012-02-08 19:31:11
Use casework
Use casework
Relativity1618
2012-02-08 19:31:11
casework
casework
DPatrick
2012-02-08 19:31:23
This looks like a job for casework. But do we have to work through all four cases?
This looks like a job for casework. But do we have to work through all four cases?
Iggy Iguana
2012-02-08 19:31:36
we only care about the 1 and 2 cases, since they are symetric to 4 and 3
we only care about the 1 and 2 cases, since they are symetric to 4 and 3
gurev
2012-02-08 19:31:36
1 and 2 is symmetrical to 3 and 4
1 and 2 is symmetrical to 3 and 4
DPatrick
2012-02-08 19:31:44
How so?
How so?
noedne
2012-02-08 19:32:03
n=1 and n=4, n=2 and n=3 are complementary
n=1 and n=4, n=2 and n=3 are complementary
RelaxationUtopia
2012-02-08 19:32:04
Chosing 2 be friends is same as 2 not vice versa
Chosing 2 be friends is same as 2 not vice versa
2718euler
2012-02-08 19:32:04
The number of ways for everyone to have 2 friends is the same as the number of ways for everyone to have 3 friends. Here's why: If everyone has 3 friends then they each have 2 friends missing.
The number of ways for everyone to have 2 friends is the same as the number of ways for everyone to have 3 friends. Here's why: If everyone has 3 friends then they each have 2 friends missing.
DPatrick
2012-02-08 19:32:26
Right! For example, suppose we have a way for n=1.
Right! For example, suppose we have a way for n=1.
DPatrick
2012-02-08 19:32:31
We just flip all the "friend"/"not friend" relationships! That is, if A and B are friends in the n=1 configuration, then they're not friends in the n=4 configuration, and vice versa.
We just flip all the "friend"/"not friend" relationships! That is, if A and B are friends in the n=1 configuration, then they're not friends in the n=4 configuration, and vice versa.
DPatrick
2012-02-08 19:32:43
We get a new arrangement in which everyone has 4 friends.
We get a new arrangement in which everyone has 4 friends.
Iggy Iguana
2012-02-08 19:32:51
so now we do casework on n=1, 2
so now we do casework on n=1, 2
noedne
2012-02-08 19:32:51
so there are only two cases!
so there are only two cases!
DPatrick
2012-02-08 19:32:58
Right, we get that (# of arrangements with n=1) = (# of arrangements with n=4), and also (# of arrangements with n=2) = (# of arrangements with n=3).
Right, we get that (# of arrangements with n=1) = (# of arrangements with n=4), and also (# of arrangements with n=2) = (# of arrangements with n=3).
DPatrick
2012-02-08 19:33:04
This means we only need count the n=1 and n=2 arrangements, and then double the total to get our answer.
This means we only need count the n=1 and n=2 arrangements, and then double the total to get our answer.
DPatrick
2012-02-08 19:33:20
Let's start with the first case -- how many ways are possible with n=1 (that is, each person has a single friend)?
Let's start with the first case -- how many ways are possible with n=1 (that is, each person has a single friend)?
theartof
2012-02-08 19:33:40
Start with 1
Start with 1
andrewjjiang97
2012-02-08 19:33:40
first choose A's friend
first choose A's friend
DPatrick
2012-02-08 19:33:54
Right, I like to count this "constructively" by actually building a valid arrangment.
Right, I like to count this "constructively" by actually building a valid arrangment.
DPatrick
2012-02-08 19:34:10
Start with A, and pick a friend for him. That's 5 choices. Those two people now have their total "friend allotment" for an n=1 arrangment, so we set them aside,
Start with A, and pick a friend for him. That's 5 choices. Those two people now have their total "friend allotment" for an n=1 arrangment, so we set them aside,
Iggy Iguana
2012-02-08 19:34:46
now there are 3 choices for someone else friend
now there are 3 choices for someone else friend
noedne
2012-02-08 19:34:46
3 remaining ways
3 remaining ways
alex31415
2012-02-08 19:34:46
3 more choices for another person
3 more choices for another person
joshy1227
2012-02-08 19:34:46
oh so for the next person theres only 3 choices
oh so for the next person theres only 3 choices
DPatrick
2012-02-08 19:35:02
Right: take a remaining person (say the first alphabetically-remaining person left), and pick a friend for him/her. That's 3 choices.
Right: take a remaining person (say the first alphabetically-remaining person left), and pick a friend for him/her. That's 3 choices.
DPatrick
2012-02-08 19:35:10
Then the two who are left must be friends.
Then the two who are left must be friends.
mdecross
2012-02-08 19:35:23
5 x 3 x 1 = 15 for n = 1 as a result
5 x 3 x 1 = 15 for n = 1 as a result
batter8642
2012-02-08 19:35:25
15 in total
15 in total
mintchip
2012-02-08 19:35:25
5*3=15
5*3=15
flyrain
2012-02-08 19:35:25
5*3*1=15
5*3*1=15
DPatrick
2012-02-08 19:35:30
So there are 5*3 = 15 arrangements where each has exactly 1 friend. That's the n=1 case.
So there are 5*3 = 15 arrangements where each has exactly 1 friend. That's the n=1 case.
DPatrick
2012-02-08 19:35:38
Next, let's count n=2 arrangements.
Next, let's count n=2 arrangements.
AlcumusGuy
2012-02-08 19:35:57
Same approach
Same approach
mathnerd101
2012-02-08 19:35:58
pick a's 2 friends?
pick a's 2 friends?
DPatrick
2012-02-08 19:36:05
Sure, we'll try the same approach. First, start at A and pick two friends. In how many ways can we do that?
Sure, we'll try the same approach. First, start at A and pick two friends. In how many ways can we do that?
anaverageaopser
2012-02-08 19:36:19
There are C(5,2) ways to choose friends for A.
There are C(5,2) ways to choose friends for A.
Hydroxide
2012-02-08 19:36:19
C(5,2) ways to choose friends for A
C(5,2) ways to choose friends for A
math-rules
2012-02-08 19:36:19
5c2=10 ways for a
5c2=10 ways for a
superbob
2012-02-08 19:36:19
for A, you have 5C2 = 10 combos of friends
for A, you have 5C2 = 10 combos of friends
DPatrick
2012-02-08 19:36:44
We have C(5,2) = 10 choices. Suppose they're B and C. (The same argument that follows works no matter which two they are.)
We have C(5,2) = 10 choices. Suppose they're B and C. (The same argument that follows works no matter which two they are.)
DPatrick
2012-02-08 19:36:49
Now what?
Now what?
Iggy Iguana
2012-02-08 19:37:15
subcase 1: b and c are friends
subcase 1: b and c are friends
pinkmuskrat
2012-02-08 19:37:15
two cases: b and c friends or not
two cases: b and c friends or not
jeff10
2012-02-08 19:37:15
B and C need to fill their other friend
B and C need to fill their other friend
andrewjjiang97
2012-02-08 19:37:18
B and C could be friends or they could be friends with other people
B and C could be friends or they could be friends with other people
DPatrick
2012-02-08 19:37:26
Exactly. We have two subcases, depending on whether B and C are themselves friends with each other.
Exactly. We have two subcases, depending on whether B and C are themselves friends with each other.
DPatrick
2012-02-08 19:37:46
If B and C are themselves friends, then ABC are all friends with each other, and DEF must also all be friends with each other. So that's 1 way to finish.
If B and C are themselves friends, then ABC are all friends with each other, and DEF must also all be friends with each other. So that's 1 way to finish.
DPatrick
2012-02-08 19:37:57
Otherwise, B and C are friends with different people. Can they be friends with the same person?
Otherwise, B and C are friends with different people. Can they be friends with the same person?
Iggy Iguana
2012-02-08 19:38:22
no because then you have 2 people left with 2 friendship pairs
no because then you have 2 people left with 2 friendship pairs
superpi83
2012-02-08 19:38:25
no. Then the last two people could only have one friend each.
no. Then the last two people could only have one friend each.
DPatrick
2012-02-08 19:38:32
No. If B and C were both friends with D, then A,B,C,D would all have 2 friends, and there wouldn't be enough left for E and F.
No. If B and C were both friends with D, then A,B,C,D would all have 2 friends, and there wouldn't be enough left for E and F.
DPatrick
2012-02-08 19:38:42
So B must be friends with one of the three remaining people, and C with a different remaining person.
So B must be friends with one of the three remaining people, and C with a different remaining person.
sparkles257
2012-02-08 19:39:05
there are 6
there are 6
PestofWest
2012-02-08 19:39:05
so there are 6 ways
so there are 6 ways
King6997
2012-02-08 19:39:05
3*2=6
3*2=6
mentalgenius
2012-02-08 19:39:05
that is 3x2 = 6
that is 3x2 = 6
DPatrick
2012-02-08 19:39:14
Right, that's 3*2 = 6 choices.
Right, that's 3*2 = 6 choices.
DPatrick
2012-02-08 19:39:29
Once we make those choices, there's only one way to finish: the person left over who still has no friends must be friends with the two most recent choices.
Once we make those choices, there's only one way to finish: the person left over who still has no friends must be friends with the two most recent choices.
BarbieRocks
2012-02-08 19:39:41
isn't it easier to think of it as a hexagon case and a two triangles case?
isn't it easier to think of it as a hexagon case and a two triangles case?
jeff10
2012-02-08 19:39:42
It could be two triangle groups of friends or one huge hexagon.
It could be two triangle groups of friends or one huge hexagon.
DPatrick
2012-02-08 19:39:58
Right: if you drew a graph of the friendships, you'd either get two triangles (in the first subcase) or a hexagon (in the second subcase).
Right: if you drew a graph of the friendships, you'd either get two triangles (in the first subcase) or a hexagon (in the second subcase).
DPatrick
2012-02-08 19:40:17
But let's finish up: how many total arrangements in our n=2 case?
But let's finish up: how many total arrangements in our n=2 case?
Iggy Iguana
2012-02-08 19:40:39
so 10(1+6)=70
so 10(1+6)=70
noedne
2012-02-08 19:40:39
10*(1+6)
10*(1+6)
superpi83
2012-02-08 19:40:39
10(6+1)=70 arrangements.
10(6+1)=70 arrangements.
ashgabat
2012-02-08 19:40:39
10*(1+6)=10*7=70
10*(1+6)=10*7=70
DPatrick
2012-02-08 19:40:53
Right, we have C(5,2) = 10 ways to pick the first two friends, then 1+6 = 7 ways to finish after we pick friends for the first person.
Right, we have C(5,2) = 10 ways to pick the first two friends, then 1+6 = 7 ways to finish after we pick friends for the first person.
DPatrick
2012-02-08 19:41:00
Hence, there are 10*7 = 70 possible arrangements for n=2.
Hence, there are 10*7 = 70 possible arrangements for n=2.
DPatrick
2012-02-08 19:41:07
And to finish the problem?
And to finish the problem?
jeff10
2012-02-08 19:41:26
15+70=85, and 85*2=170.
15+70=85, and 85*2=170.
Relativity1618
2012-02-08 19:41:27
so 2(70+15)=(B) 170
so 2(70+15)=(B) 170
ashgabat
2012-02-08 19:41:27
2*(70+15)=170
2*(70+15)=170
eshoyfer
2012-02-08 19:41:27
70+15 = 85, then double that.
70+15 = 85, then double that.
DPatrick
2012-02-08 19:41:30
We had a count of 15 for n=1 and 70 for n=2.
We had a count of 15 for n=1 and 70 for n=2.
DPatrick
2012-02-08 19:41:37
Thus our final answer is 2(15+70) = 2(85) = 170. Answer (B).
Thus our final answer is 2(15+70) = 2(85) = 170. Answer (B).
DPatrick
2012-02-08 19:42:05
Don't forget to double at the end to capture the n=3 and n=4 cases that we skipped? (Of course, 85 isn't an answer choice.)
Don't forget to double at the end to capture the n=3 and n=4 cases that we skipped? (Of course, 85 isn't an answer choice.)
DPatrick
2012-02-08 19:42:30
Let's continue with #24 on the AMC 10A, which is also #21 on the 12A:
Let's continue with #24 on the AMC 10A, which is also #21 on the 12A:
DPatrick
2012-02-08 19:42:52
Yikes. These look ugly. What can we do?
Yikes. These look ugly. What can we do?
DPatrick
2012-02-08 19:43:18
Many many of you said "add the equations together", which is the right thing to do.
Many many of you said "add the equations together", which is the right thing to do.
DPatrick
2012-02-08 19:43:38
If you don't see this right away, you still have to try something. Don't just stare blankly at a problem -- try something!
If you don't see this right away, you still have to try something. Don't just stare blankly at a problem -- try something!
DPatrick
2012-02-08 19:43:41
Ideally we'd like to have something we can factor on the left side.
Ideally we'd like to have something we can factor on the left side.
DPatrick
2012-02-08 19:43:56
Adding them together is at least something worth trying if you don't see right away that it will help.
Adding them together is at least something worth trying if you don't see right away that it will help.
noedne
2012-02-08 19:44:32
(a-b)^2+(b-c)^2+(c-a)^2=14
(a-b)^2+(b-c)^2+(c-a)^2=14
xjava
2012-02-08 19:44:33
(a-b)^2+(b-c)^2+(c-a)^2=14
(a-b)^2+(b-c)^2+(c-a)^2=14
lucylai
2012-02-08 19:44:33
you get (a-b)^2+(b-c)^2+(a-c)^2=14
you get (a-b)^2+(b-c)^2+(a-c)^2=14
DPatrick
2012-02-08 19:44:38
Happy day, this factors!
Happy day, this factors!
DPatrick
2012-02-08 19:44:56
Note how we wrote this so that the term inside each set of parentheses is a nonnegative integer.
Note how we wrote this so that the term inside each set of parentheses is a nonnegative integer.
DPatrick
2012-02-08 19:45:06
How can we write 14 as the sum of three nonnegative perfect squares?
How can we write 14 as the sum of three nonnegative perfect squares?
lucylai
2012-02-08 19:45:29
the pairs have to be 1, 9, and 4
the pairs have to be 1, 9, and 4
Watermelon876
2012-02-08 19:45:30
14=1+9+4
14=1+9+4
number.sense
2012-02-08 19:45:30
they must be 1, 4, 9'
they must be 1, 4, 9'
yankeefan6795
2012-02-08 19:45:30
3^2+2^2+1^2=14
3^2+2^2+1^2=14
DPatrick
2012-02-08 19:45:36
There's only one way: 1 + 4 + 9 = 14.
There's only one way: 1 + 4 + 9 = 14.
DPatrick
2012-02-08 19:45:45
What can we conclude?
What can we conclude?
jeff10
2012-02-08 19:46:10
a-c has to be 3 because it is 3^2+2^2+1^2 and 3 is the biggest of 1, 2, and 3.
a-c has to be 3 because it is 3^2+2^2+1^2 and 3 is the biggest of 1, 2, and 3.
Hydroxide
2012-02-08 19:46:10
a-b, a-c, and b-c have to 1,2, or 3
a-b, a-c, and b-c have to 1,2, or 3
AceOfDiamonds
2012-02-08 19:46:10
a-c = 3
a-c = 3
mcdonalds106_7
2012-02-08 19:46:10
a-c is the biggest, so (a-c)^2=9!
a-c is the biggest, so (a-c)^2=9!
DPatrick
2012-02-08 19:46:25
We must have a-c = 3 since that's the biggest difference, and one each of a-b and b-c must be 1 or 2. We can't tell which is which though.
We must have a-c = 3 since that's the biggest difference, and one each of a-b and b-c must be 1 or 2. We can't tell which is which though.
Iggy Iguana
2012-02-08 19:46:39
casework
casework
xjava
2012-02-08 19:46:40
try both cases
try both cases
DPatrick
2012-02-08 19:46:46
Yep, we're back to casework.
Yep, we're back to casework.
number.sense
2012-02-08 19:47:19
plug into the top equation
plug into the top equation
yankeefan6795
2012-02-08 19:47:19
plug into first original equation
plug into first original equation
scgorantla
2012-02-08 19:47:19
we plug both cases in to the original equation
we plug both cases in to the original equation
mcdonalds106_7
2012-02-08 19:47:19
plug it into the first equation!
plug it into the first equation!
DPatrick
2012-02-08 19:47:24
We plug these into one of the equations and solve for a. My choice would be to use the first equation, since it's a lot simpler.
We plug these into one of the equations and solve for a. My choice would be to use the first equation, since it's a lot simpler.
DPatrick
2012-02-08 19:47:37
Now it's just algebra crunching.
Now it's just algebra crunching.
PestofWest
2012-02-08 19:48:17
2021/7
2021/7
Relativity1618
2012-02-08 19:48:17
2021 is not divisible by 7
2021 is not divisible by 7
DPatrick
2012-02-08 19:48:24
This gives 7a = 2021, but then a = 2021/7, which is not an integer. So no solution in this case.
This gives 7a = 2021, but then a = 2021/7, which is not an integer. So no solution in this case.
DPatrick
2012-02-08 19:48:32
The other case had better work!
The other case had better work!
DPatrick
2012-02-08 19:49:11
This gives 8a = 2024, so a = 253. Answer (E).
This gives 8a = 2024, so a = 253. Answer (E).
DPatrick
2012-02-08 19:49:39
The "clever" step was adding the equations and recognizing the factorization.
The "clever" step was adding the equations and recognizing the factorization.
DPatrick
2012-02-08 19:50:00
Recognizing when something factors is not always easy, but is a skill that gets better with practice and experience.
Recognizing when something factors is not always easy, but is a skill that gets better with practice and experience.
DPatrick
2012-02-08 19:50:29
Let finish up the end of the AMC 10A with #25:
Let finish up the end of the AMC 10A with #25:
DPatrick
2012-02-08 19:50:55
In general, how do we calculate this sort of probability, when there are continuously-many infinite possibilities?
In general, how do we calculate this sort of probability, when there are continuously-many infinite possibilities?
lucylai
2012-02-08 19:51:17
geometric
geometric
Lalagato
2012-02-08 19:51:17
geometry!
geometry!
yankeefan6795
2012-02-08 19:51:17
geometrically
geometrically
alberthyang
2012-02-08 19:51:17
geometric probability?
geometric probability?
talkinaway
2012-02-08 19:51:17
Areas and/or volumes
Areas and/or volumes
DPatrick
2012-02-08 19:51:51
The denominator is easy: what is the "possible" region?
The denominator is easy: what is the "possible" region?
scgorantla
2012-02-08 19:52:08
n^3
n^3
number.sense
2012-02-08 19:52:08
n^3
n^3
Lalagato
2012-02-08 19:52:08
n^3
n^3
Binomial-theorem
2012-02-08 19:52:08
n^3
n^3
BarbieRocks
2012-02-08 19:52:08
n^3
n^3
hyperbolictangent
2012-02-08 19:52:08
A cube of side n
A cube of side n
scgorantla
2012-02-08 19:52:08
cube with side length of n
cube with side length of n
DPatrick
2012-02-08 19:52:36
The successful region is trickier. (Obviously, or this wouldn't be a #25!)
The successful region is trickier. (Obviously, or this wouldn't be a #25!)
DPatrick
2012-02-08 19:52:45
What can we do to simplify its computation?
What can we do to simplify its computation?
Binomial-theorem
2012-02-08 19:53:16
WLOG let x>y>z
WLOG let x>y>z
noedne
2012-02-08 19:53:16
0<=x<=y<=z<=n
0<=x<=y<=z<=n
BarbieRocks
2012-02-08 19:53:16
assume x>=y>=z and multiply by 6 later
assume x>=y>=z and multiply by 6 later
GoldenFrog1618
2012-02-08 19:53:16
assume x>y>z, and then multiply by 6
assume x>y>z, and then multiply by 6
DPatrick
2012-02-08 19:53:29
Right. The "successful" region is actually made up of six identical pieces, depending on the order of x, y, and z.
Right. The "successful" region is actually made up of six identical pieces, depending on the order of x, y, and z.
DPatrick
2012-02-08 19:53:40
Otherwise these regions have identical shapes, so if we find the volume of one of them, we can multiply by 6 to get our numerator.
Otherwise these regions have identical shapes, so if we find the volume of one of them, we can multiply by 6 to get our numerator.
DPatrick
2012-02-08 19:53:53
So let's focus on x < y < z.
So let's focus on x < y < z.
DPatrick
2012-02-08 19:53:58
What is the condition for "success"?
What is the condition for "success"?
lucylai
2012-02-08 19:54:35
z-y>1, y-x>1
z-y>1, y-x>1
memphis_tiger
2012-02-08 19:54:35
z>y+1, y>x+1
z>y+1, y>x+1
Binomial-theorem
2012-02-08 19:54:35
z-y>1 and y-x>1
z-y>1 and y-x>1
DPatrick
2012-02-08 19:54:55
You could actually determine this region directly and compute its volume, but what's a slick way to compute the volume of this region?
You could actually determine this region directly and compute its volume, but what's a slick way to compute the volume of this region?
pl210741
2012-02-08 19:55:44
Its a pyramid
Its a pyramid
Iggy Iguana
2012-02-08 19:55:44
transformation
transformation
DPatrick
2012-02-08 19:55:58
Indeed, it is a pyramid, and you could directly determine its base and its height.
Indeed, it is a pyramid, and you could directly determine its base and its height.
DPatrick
2012-02-08 19:56:06
But there is a clever transformation that makes it easy to compute.
But there is a clever transformation that makes it easy to compute.
ClassClown
2012-02-08 19:56:32
You just move the base from the original one
You just move the base from the original one
DPatrick
2012-02-08 19:56:51
That's exactly what this is doing: it's just sliding the region 1 unit in the y-direction and 2 units in the z-direction.
That's exactly what this is doing: it's just sliding the region 1 unit in the y-direction and 2 units in the z-direction.
DPatrick
2012-02-08 19:57:12
But now it's clear that what we have is just the part of the cube of side length (n-2) with x' <= y' <= z'.
But now it's clear that what we have is just the part of the cube of side length (n-2) with x' <= y' <= z'.
DPatrick
2012-02-08 19:57:37
This is 1/6 of the cube of side length (n-2).
This is 1/6 of the cube of side length (n-2).
AkshajK
2012-02-08 19:57:59
SO WHEN YOU MULTIPLY BY SIX YOU GET (n-2)^3 / n^3
SO WHEN YOU MULTIPLY BY SIX YOU GET (n-2)^3 / n^3
DPatrick
2012-02-08 19:58:24
Right. There are 6 pieces (for the 6 different orderings of x, y, and z), and each piece is 1/6 of the cube of side length (n-2).
Right. There are 6 pieces (for the 6 different orderings of x, y, and z), and each piece is 1/6 of the cube of side length (n-2).
jeff10
2012-02-08 19:58:57
Just keep testing until you get that fraction is greater than 1/2.
Just keep testing until you get that fraction is greater than 1/2.
yankeefan6795
2012-02-08 19:59:05
which needs to be greater than 1/2
which needs to be greater than 1/2
Relativity1618
2012-02-08 19:59:08
that must exceed 1/2
that must exceed 1/2
alex31415
2012-02-08 19:59:36
n=10 will lead to 0.512, which is greater than 1/2. So 10(D) is the answer.
n=10 will lead to 0.512, which is greater than 1/2. So 10(D) is the answer.
Relativity1618
2012-02-08 19:59:36
so n=10
so n=10
pieofdoom51413
2012-02-08 19:59:36
n = 10
n = 10
kaiz99
2012-02-08 19:59:36
n=10
n=10
number.sense
2012-02-08 19:59:36
9 is not big enough
9 is not big enough
DPatrick
2012-02-08 19:59:41
Note that (0.8)^3 = 0.512 > 0.5, so n=10 definitely works.
Note that (0.8)^3 = 0.512 > 0.5, so n=10 definitely works.
DPatrick
2012-02-08 19:59:52
We check n=9: (7/9)^3 = 343/729 < 1/2. So n=9 doesn't work.
We check n=9: (7/9)^3 = 343/729 < 1/2. So n=9 doesn't work.
DPatrick
2012-02-08 19:59:59
Hence the smallest n that works is n=10. Answer (D).
Hence the smallest n that works is n=10. Answer (D).
DPatrick
2012-02-08 20:00:31
So that wraps up the last five problems on the AMC 10A.
So that wraps up the last five problems on the AMC 10A.
DPatrick
2012-02-08 20:00:38
Now let's go over to the 12A!
Now let's go over to the 12A!
DPatrick
2012-02-08 20:00:59
We already did #21: it was #24 on the 10A as well (the (a-b)^2+... problem).
We already did #21: it was #24 on the 10A as well (the (a-b)^2+... problem).
DPatrick
2012-02-08 20:01:05
So we'll pick up with #22:
So we'll pick up with #22:
DPatrick
2012-02-08 20:01:32
What the heck is going on here?
What the heck is going on here?
AlcumusGuy
2012-02-08 20:01:40
What does the big U mean?
What does the big U mean?
alex31415
2012-02-08 20:01:40
What does that big "U" mean?
What does that big "U" mean?
DPatrick
2012-02-08 20:01:50
It's "union". It means take all the planes together.
It's "union". It means take all the planes together.
Watermelon876
2012-02-08 20:02:00
Apparently a cube is being cut up. the u is the sum of all the planes by the sum axiom
Apparently a cube is being cut up. the u is the sum of all the planes by the sum axiom
mdecross
2012-02-08 20:02:02
Draw a pretty picture
Draw a pretty picture
DPatrick
2012-02-08 20:02:27
What's going on in this problem is buried under a lot of notation.
What's going on in this problem is buried under a lot of notation.
2718euler
2012-02-08 20:02:36
The planes must intersect the cube ONLY at the segments joining the midpoints of pairs of edges, or else the intersection would include some other points.
The planes must intersect the cube ONLY at the segments joining the midpoints of pairs of edges, or else the intersection would include some other points.
DPatrick
2012-02-08 20:02:47
Right. We're chopping a cube with a bunch of planes. The places where the planes cut into the cube have to be these special segments.
Right. We're chopping a cube with a bunch of planes. The places where the planes cut into the cube have to be these special segments.
DPatrick
2012-02-08 20:03:01
The blue lines are the segments that we have to hit.
The blue lines are the segments that we have to hit.
DPatrick
2012-02-08 20:03:15
Notice there are 6 segments on each face, for a total of 6*6 = 36 segments that we have to hit.
Notice there are 6 segments on each face, for a total of 6*6 = 36 segments that we have to hit.
DPatrick
2012-02-08 20:03:28
So how do we attack this problem?
So how do we attack this problem?
2718euler
2012-02-08 20:03:45
What are the different kinds of planes?
What are the different kinds of planes?
number.sense
2012-02-08 20:03:46
what kinds of planes can we have is the question
what kinds of planes can we have is the question
DPatrick
2012-02-08 20:04:04
Indeed. Let's start with trying to find the maximum, because that's doesn't require quite as much cleverness -- we just have to determine how many possible planes there are. (And, determining all the possible planes will probably help us with the minimum argument too.)
Indeed. Let's start with trying to find the maximum, because that's doesn't require quite as much cleverness -- we just have to determine how many possible planes there are. (And, determining all the possible planes will probably help us with the minimum argument too.)
DPatrick
2012-02-08 20:04:43
There are probably lots of ways to go about this, but the key is to be methodical and organized, and have a plan. Don't start aimlessly counting planes without a plan. (You can't spell "plane" with "plan", after all.)
There are probably lots of ways to go about this, but the key is to be methodical and organized, and have a plan. Don't start aimlessly counting planes without a plan. (You can't spell "plane" with "plan", after all.)
fmasroor
2012-02-08 20:04:55
without
without
DPatrick
2012-02-08 20:05:03
Yeah, I ruined my own joke. :(
Yeah, I ruined my own joke. :(
2718euler
2012-02-08 20:05:22
What kinds of segments do the planes go through?
What kinds of segments do the planes go through?
DPatrick
2012-02-08 20:05:34
That's a good idea. What I did was focus on a single point:
That's a good idea. What I did was focus on a single point:
DPatrick
2012-02-08 20:05:44
(I circled it above)
(I circled it above)
DPatrick
2012-02-08 20:05:51
How many planes (satisfying the conditions of the problem) pass through this point?
How many planes (satisfying the conditions of the problem) pass through this point?
ziv
2012-02-08 20:06:25
9: pick one line on top face, one on front face
9: pick one line on top face, one on front face
DPatrick
2012-02-08 20:06:29
Right!
Right!
DPatrick
2012-02-08 20:06:40
Each valid plane through the circled point must contain one of the 3 segments passing through the circled point and lying on the top face.
Each valid plane through the circled point must contain one of the 3 segments passing through the circled point and lying on the top face.
DPatrick
2012-02-08 20:06:49
Each valid plane must also contain one of the 3 segments passing through the circled point and lying on the front face.
Each valid plane must also contain one of the 3 segments passing through the circled point and lying on the front face.
DPatrick
2012-02-08 20:07:00
So there are 3*3 = 9 possible planes through the circled point.
So there are 3*3 = 9 possible planes through the circled point.
DPatrick
2012-02-08 20:07:32
And we know that there are 12 points like the circled one (one per edge of the cube), so does that mean there are 9*12 = 108 possible planes?
And we know that there are 12 points like the circled one (one per edge of the cube), so does that mean there are 9*12 = 108 possible planes?
hyperbolictangent
2012-02-08 20:07:49
Overcounting
Overcounting
number.sense
2012-02-08 20:07:49
NO! We're vastly overcounting
NO! We're vastly overcounting
fro116
2012-02-08 20:07:49
no, we are overcounting
no, we are overcounting
2718euler
2012-02-08 20:07:49
NO! We've overcounted
NO! We've overcounted
DPatrick
2012-02-08 20:07:58
Indeed, since all those planes pass through multiple points, so each plane is counted multiple times in our dumb count of 108.
Indeed, since all those planes pass through multiple points, so each plane is counted multiple times in our dumb count of 108.
DPatrick
2012-02-08 20:08:19
How much each plane is overcounted depends on the type of plane, meaning it depending on the choices of segments that we make.
How much each plane is overcounted depends on the type of plane, meaning it depending on the choices of segments that we make.
number.sense
2012-02-08 20:08:41
we can't evenly divide that number either, because some planes pass thorugh 4 or 3
we can't evenly divide that number either, because some planes pass thorugh 4 or 3
DPatrick
2012-02-08 20:08:49
Right...and maybe some other number too...
Right...and maybe some other number too...
DPatrick
2012-02-08 20:09:07
For example, we might choose a plane using two of the shorter "diagonal" segments on the same side, like this:
For example, we might choose a plane using two of the shorter "diagonal" segments on the same side, like this:
DPatrick
2012-02-08 20:09:33
How many our of special points does that plane pass through?
How many our of special points does that plane pass through?
number.sense
2012-02-08 20:09:47
3
3
mathnerd101
2012-02-08 20:09:47
3
3
Flamel
2012-02-08 20:09:47
3
3
VIPMaster
2012-02-08 20:09:47
3
3
Asmodeus1123
2012-02-08 20:09:47
3
3
yankeefan6795
2012-02-08 20:09:47
3
3
mathlover3737
2012-02-08 20:09:47
3
3
sparkles257
2012-02-08 20:09:47
3
3
DPatrick
2012-02-08 20:10:00
Right, 3. (Basically the three that we can see in the above picture.)
Right, 3. (Basically the three that we can see in the above picture.)
DPatrick
2012-02-08 20:10:14
And how many of the 9 planes through the circled point are of this type?
And how many of the 9 planes through the circled point are of this type?
mathnerd101
2012-02-08 20:10:36
2
2
yankeefan6795
2012-02-08 20:10:36
2
2
Flamel
2012-02-08 20:10:36
2
2
Iggy Iguana
2012-02-08 20:10:36
2.
2.
mathlover3737
2012-02-08 20:10:36
2
2
Shoelace Thm.
2012-02-08 20:10:36
2
2
DPatrick
2012-02-08 20:10:49
There are 2 of this type of plane at the circled point, one on each side.
There are 2 of this type of plane at the circled point, one on each side.
DPatrick
2012-02-08 20:11:16
So each circled point has 2 planes of this type, and there are 12 of these points (one on each edge), so that's a preliminary overcount of 2*12 = 24.
So each circled point has 2 planes of this type, and there are 12 of these points (one on each edge), so that's a preliminary overcount of 2*12 = 24.
mathlover3737
2012-02-08 20:11:40
24/3=8 of these
24/3=8 of these
DPatrick
2012-02-08 20:11:57
But we just said that each of these planes passes through 3 points, so it gets counted 3 times.
But we just said that each of these planes passes through 3 points, so it gets counted 3 times.
DPatrick
2012-02-08 20:12:14
So there are a total of 24/3 = 8 planes of this type (that intersect 3 of our special points).
So there are a total of 24/3 = 8 planes of this type (that intersect 3 of our special points).
DPatrick
2012-02-08 20:12:46
Another type of plane through the circled points uses a shorter diagonal segment, but on opposite sides of the circled point:
Another type of plane through the circled points uses a shorter diagonal segment, but on opposite sides of the circled point:
DPatrick
2012-02-08 20:13:05
How many of our special points does this intersect?
How many of our special points does this intersect?
number.sense
2012-02-08 20:13:28
6 points
6 points
leekspeak
2012-02-08 20:13:28
6
6
mathnerd101
2012-02-08 20:13:28
6
6
mdecross
2012-02-08 20:13:28
6
6
ChipDale
2012-02-08 20:13:28
6
6
x31415926535x
2012-02-08 20:13:28
6
6
DPatrick
2012-02-08 20:13:37
The plane intersects the cube in a hexagon. (There are two more intersection points that we can't see in the back of the cube, and there's one side of the hexagon on each of the six faces of the cube.)
The plane intersects the cube in a hexagon. (There are two more intersection points that we can't see in the back of the cube, and there's one side of the hexagon on each of the six faces of the cube.)
DPatrick
2012-02-08 20:14:00
And again, there are 2 of these through each circled point, one in each direction.
And again, there are 2 of these through each circled point, one in each direction.
DPatrick
2012-02-08 20:14:08
So in total, how many planes of this type are there?
So in total, how many planes of this type are there?
Iggy Iguana
2012-02-08 20:14:33
so we have 24/6=4
so we have 24/6=4
math-rules
2012-02-08 20:14:33
12*2/6=4
12*2/6=4
number.sense
2012-02-08 20:14:33
24/6 = 4
24/6 = 4
proglote
2012-02-08 20:14:33
24/6 = 4 planes
24/6 = 4 planes
DPatrick
2012-02-08 20:14:45
Right: we have a preliminary overcount of 2*12 = 24 planes, but each passes through 6 points, so there are 24/6 = 4 of them.
Right: we have a preliminary overcount of 2*12 = 24 planes, but each passes through 6 points, so there are 24/6 = 4 of them.
DPatrick
2012-02-08 20:15:01
That leaves 9-2-2=5 choices for a pair of segments at the circled points; each of these choices contains one or two of the longer segments (parallel to the edges of the cube).
That leaves 9-2-2=5 choices for a pair of segments at the circled points; each of these choices contains one or two of the longer segments (parallel to the edges of the cube).
DPatrick
2012-02-08 20:15:30
I give you a sec to digest these new pics...how many special points do these red planes intersect the cube at?
I give you a sec to digest these new pics...how many special points do these red planes intersect the cube at?
Iggy Iguana
2012-02-08 20:16:01
each one passes through 4 poitns
each one passes through 4 poitns
number.sense
2012-02-08 20:16:02
4 special points
4 special points
yankeefan6795
2012-02-08 20:16:02
4 each
4 each
Rocksolid
2012-02-08 20:16:02
4 both
4 both
DPatrick
2012-02-08 20:16:15
They come in two types, but either type intersects 4 points.
They come in two types, but either type intersects 4 points.
number.sense
2012-02-08 20:16:27
5*12/4 = 15 total planes
5*12/4 = 15 total planes
Iggy Iguana
2012-02-08 20:16:27
so 60/4=15
so 60/4=15
DPatrick
2012-02-08 20:16:43
Right. There are 5 planes at each of the 12 points, and each passes through 4 points. So there are 5*12/4 = 15 of them.
Right. There are 5 planes at each of the 12 points, and each passes through 4 points. So there are 5*12/4 = 15 of them.
supermathman
2012-02-08 20:16:58
so max is gone be 27
so max is gone be 27
number.sense
2012-02-08 20:16:58
27 seems to be the maximum then
27 seems to be the maximum then
DPatrick
2012-02-08 20:17:09
We've methodically counted all the planes. We add the cases: 8 + 4 + 15 = 27 planes.
We've methodically counted all the planes. We add the cases: 8 + 4 + 15 = 27 planes.
DPatrick
2012-02-08 20:17:16
These are all distinct and are all the legal planes, so 27 is the maximum number of planes.
These are all distinct and are all the legal planes, so 27 is the maximum number of planes.
DPatrick
2012-02-08 20:17:58
That was just methodical casework, and while it may have taken some geometric insight, it didn't require any special degree of "cleverness", just an organized plan.
That was just methodical casework, and while it may have taken some geometric insight, it didn't require any special degree of "cleverness", just an organized plan.
DPatrick
2012-02-08 20:18:03
Now for the more subtle question: what is the minimum number that we need?
Now for the more subtle question: what is the minimum number that we need?
DPatrick
2012-02-08 20:18:19
There are a total of 36 segments that we need to hit (6 on each side).
There are a total of 36 segments that we need to hit (6 on each side).
DPatrick
2012-02-08 20:18:24
How can we hit all 36 segments most efficiently?
How can we hit all 36 segments most efficiently?
mathlover3737
2012-02-08 20:18:30
use the 6-point planes
use the 6-point planes
number.sense
2012-02-08 20:18:36
now for the minimum; it seems like the hexagons can get a lot of bang for their buck :)
now for the minimum; it seems like the hexagons can get a lot of bang for their buck :)
Iggy Iguana
2012-02-08 20:18:36
using hexagons
using hexagons
DPatrick
2012-02-08 20:19:02
Indeed, using the 4 hexagon planes will give us 6 segments each, so we'll get a total of 24 segments -- all the "diagonal" segments in fact.
Indeed, using the 4 hexagon planes will give us 6 segments each, so we'll get a total of 24 segments -- all the "diagonal" segments in fact.
DPatrick
2012-02-08 20:19:10
Here's the hexagon pic again:
Here's the hexagon pic again:
DPatrick
2012-02-08 20:19:37
So we're left with just the segments parallel to the edges of the cube.
So we're left with just the segments parallel to the edges of the cube.
mdecross
2012-02-08 20:19:57
For the min, you can quickly eliminate the + on every face with only 3 planes used
For the min, you can quickly eliminate the + on every face with only 3 planes used
314159Nerd
2012-02-08 20:19:57
3 more planes for vertical and horizontal
3 more planes for vertical and horizontal
yankeefan6795
2012-02-08 20:19:57
3 more!
3 more!
superpi83
2012-02-08 20:19:57
3 planes to cover the parallel segments.
3 planes to cover the parallel segments.
Iggy Iguana
2012-02-08 20:19:57
we can use 3 planes parallel to faces
we can use 3 planes parallel to faces
DPatrick
2012-02-08 20:20:06
Right, we need the 3 planes that look like this:
Right, we need the 3 planes that look like this:
DPatrick
2012-02-08 20:20:20
Using the three planes that are parallel to the sides of the cube will give all 12 longer segments.
Using the three planes that are parallel to the sides of the cube will give all 12 longer segments.
number.sense
2012-02-08 20:20:31
so 7 is the minimum
so 7 is the minimum
Iggy Iguana
2012-02-08 20:20:31
so that's 7 in total
so that's 7 in total
DPatrick
2012-02-08 20:20:35
So 7 planes is the minimum.
So 7 planes is the minimum.
number.sense
2012-02-08 20:20:45
and 27-7 = (C) is our answer
and 27-7 = (C) is our answer
yankeefan6795
2012-02-08 20:20:45
27-7=20
27-7=20
Cortana
2012-02-08 20:20:45
27-7=20
27-7=20
Iggy Iguana
2012-02-08 20:20:45
the answer is 27-7=20 (C)
the answer is 27-7=20 (C)
DPatrick
2012-02-08 20:20:48
Thus the answer is 27 - 7 = 20. Answer (C).
Thus the answer is 27 - 7 = 20. Answer (C).
DPatrick
2012-02-08 20:21:26
I liked this problem -- it was a nice blend of geometric intuition and careful counting with overcounting.
I liked this problem -- it was a nice blend of geometric intuition and careful counting with overcounting.
DPatrick
2012-02-08 20:21:54
(I didn't particularly like the way the question was phrased though -- I think it could have probably been phrased more simply.)
(I didn't particularly like the way the question was phrased though -- I think it could have probably been phrased more simply.)
DPatrick
2012-02-08 20:22:01
Anyway, on to #23:
Anyway, on to #23:
Asmodeus1123
2012-02-08 20:22:48
picture?
picture?
Iggy Iguana
2012-02-08 20:22:48
draw the square
draw the square
DPatrick
2012-02-08 20:22:52
We can sketch a picture:
We can sketch a picture:
DPatrick
2012-02-08 20:23:10
S is the square in bold.
S is the square in bold.
DPatrick
2012-02-08 20:23:20
Now what?
Now what?
supermathman
2012-02-08 20:23:45
consider the unit square U with vertices 0,0 1,0 1,1 0,1
consider the unit square U with vertices 0,0 1,0 1,1 0,1
number.sense
2012-02-08 20:23:57
how can we cover 2
how can we cover 2
Shoelace Thm.
2012-02-08 20:23:57
FInd conditions on how it can contain two lattice points
FInd conditions on how it can contain two lattice points
Iggy Iguana
2012-02-08 20:23:57
it can only contain 2 adjacent lattice points at the most
it can only contain 2 adjacent lattice points at the most
DPatrick
2012-02-08 20:24:09
There are a few simplifying observations we can make.
There are a few simplifying observations we can make.
DPatrick
2012-02-08 20:24:22
The 2012 seems irrelevant, and in fact it is. By symmetry, we can assume 0 <= x <= 1 and 0 <= y <= 1, or indeed we can just look at any lattice square.
The 2012 seems irrelevant, and in fact it is. By symmetry, we can assume 0 <= x <= 1 and 0 <= y <= 1, or indeed we can just look at any lattice square.
DPatrick
2012-02-08 20:25:07
You might notice that the square S has area 1 and that its sides have slope 3/4 and -4/3.
You might notice that the square S has area 1 and that its sides have slope 3/4 and -4/3.
DPatrick
2012-02-08 20:25:14
Let's just tuck that factoid away for later.
Let's just tuck that factoid away for later.
DPatrick
2012-02-08 20:25:42
You might also notice that if translation of our square covers two lattice points, they have to be adjacent. (Any two non-adjacent points are too far apart.)
You might also notice that if translation of our square covers two lattice points, they have to be adjacent. (Any two non-adjacent points are too far apart.)
DPatrick
2012-02-08 20:26:21
So let's look at the unit square with lower-left corner (0,0), and find the translations of S that cover both the points (0,0) and (1,0).
(By symmetry, there will be 3 equal regions covering the other three adjacent pairs of corners.)
So let's look at the unit square with lower-left corner (0,0), and find the translations of S that cover both the points (0,0) and (1,0).
(By symmetry, there will be 3 equal regions covering the other three adjacent pairs of corners.)
DPatrick
2012-02-08 20:26:58
Here's the question that breaks open the problem: where must the center of S be in order for S to contain the point (0,0)?
Here's the question that breaks open the problem: where must the center of S be in order for S to contain the point (0,0)?
Cortana
2012-02-08 20:27:42
Inside the square?
Inside the square?
DPatrick
2012-02-08 20:28:07
Right. To say it in the technical language of the problem, v must be in S for T(v) to contain (0,0).
Right. To say it in the technical language of the problem, v must be in S for T(v) to contain (0,0).
DPatrick
2012-02-08 20:28:36
To say it more intuitively, we can slide S so that it's centered at any point that's inside the original S.
To say it more intuitively, we can slide S so that it's centered at any point that's inside the original S.
DPatrick
2012-02-08 20:28:51
This will still keep the origin (0,0) inside of the slid square.
This will still keep the origin (0,0) inside of the slid square.
DPatrick
2012-02-08 20:29:20
In other words, the center of S (after we move it) must be in the red square below. Note the red square is the same as the original square!
In other words, the center of S (after we move it) must be in the red square below. Note the red square is the same as the original square!
DPatrick
2012-02-08 20:29:47
Similarly, the blue square above is the locations at which the center of the tilted square captures the point (1,0).
Similarly, the blue square above is the locations at which the center of the tilted square captures the point (1,0).
DPatrick
2012-02-08 20:30:23
So the purple area (where the tilted squares overlap) is the region where we can center the tilted square and get both (0,0) and (1,0).
So the purple area (where the tilted squares overlap) is the region where we can center the tilted square and get both (0,0) and (1,0).
DPatrick
2012-02-08 20:30:36
(I admit this is a subtle argument. You may have to think about it more later on your own.)
(I admit this is a subtle argument. You may have to think about it more later on your own.)
Iggy Iguana
2012-02-08 20:30:46
find area of overlap
find area of overlap
brokenfixer
2012-02-08 20:30:50
two purple rectangles total
two purple rectangles total
number.sense
2012-02-08 20:31:03
2 times that purple area / white area is what we want
2 times that purple area / white area is what we want
DPatrick
2012-02-08 20:31:28
Exactly. We only want to count what works that inside the unit square, so we only want half of that purple region to capture the bottom two points.
Exactly. We only want to count what works that inside the unit square, so we only want half of that purple region to capture the bottom two points.
DPatrick
2012-02-08 20:31:48
But there are four such regions (one on each side of the square, capturing the two points on that side). Thus, 4 times half of the purple area is our answer. (That is, twice its area is our answer.)
But there are four such regions (one on each side of the square, capturing the two points on that side). Thus, 4 times half of the purple area is our answer. (That is, twice its area is our answer.)
saprmarks
2012-02-08 20:31:57
is this where we use the slopes?
is this where we use the slopes?
DPatrick
2012-02-08 20:32:16
Indeed, this is where we use the slopes! There's a really easy way to find the area of the purple rectangle if we are clever.
Indeed, this is where we use the slopes! There's a really easy way to find the area of the purple rectangle if we are clever.
DPatrick
2012-02-08 20:32:42
We can get all coordinate-y and compute the actual points and bash from there, but there's a vastly simpler way.
We can get all coordinate-y and compute the actual points and bash from there, but there's a vastly simpler way.
DPatrick
2012-02-08 20:33:28
Let me shade a particularly interesting part of the picture:
Let me shade a particularly interesting part of the picture:
Yongyi781
2012-02-08 20:33:31
0.6-0.8-1.0 "hole" at bottom
0.6-0.8-1.0 "hole" at bottom
DPatrick
2012-02-08 20:33:34
Exactly!
Exactly!
DPatrick
2012-02-08 20:34:12
Because we know the slopes, we know that the grey triangle is similar to a 3-4-5 right triangle!
Because we know the slopes, we know that the grey triangle is similar to a 3-4-5 right triangle!
DPatrick
2012-02-08 20:34:23
So since the hypotenuse has length 1, its red side has length 0.6 and its blue side has length 0.8.
So since the hypotenuse has length 1, its red side has length 0.6 and its blue side has length 0.8.
Iggy Iguana
2012-02-08 20:34:33
so the sides of rectangle are .2 and .4
so the sides of rectangle are .2 and .4
DPatrick
2012-02-08 20:34:45
Yes! The purple rectangle now has sides of length 0.4 (the continuation of the red square) and 0.2 (the continuation of the blue square).
Yes! The purple rectangle now has sides of length 0.4 (the continuation of the red square) and 0.2 (the continuation of the blue square).
DPatrick
2012-02-08 20:34:55
So its area is (0.4)(0.2) = 0.08.
So its area is (0.4)(0.2) = 0.08.
Draco
2012-02-08 20:35:06
Therefore .2*.4*2=.16.
Therefore .2*.4*2=.16.
Iggy Iguana
2012-02-08 20:35:06
so 2*0.2*0.4=0.16 (C)
so 2*0.2*0.4=0.16 (C)
number.sense
2012-02-08 20:35:06
so 0.08 is the area of one rectangle, giving 0.16 (C)
so 0.08 is the area of one rectangle, giving 0.16 (C)
DPatrick
2012-02-08 20:35:10
Hence our final answer is 2*0.08 = 0.16. Answer (C).
Hence our final answer is 2*0.08 = 0.16. Answer (C).
DPatrick
2012-02-08 20:35:31
This was pretty hard I thought.
This was pretty hard I thought.
DPatrick
2012-02-08 20:35:44
But I liked its pretty solution at the end.
But I liked its pretty solution at the end.
DPatrick
2012-02-08 20:36:12
My editorial comment is that I thought 22-25 were harder than usual, but I thought that 15-20 (or so) were easier than usual, so it kind of balances out.
My editorial comment is that I thought 22-25 were harder than usual, but I thought that 15-20 (or so) were easier than usual, so it kind of balances out.
DPatrick
2012-02-08 20:36:28
Let's press on with #24:
Let's press on with #24:
DPatrick
2012-02-08 20:36:45
Double yikes!
Double yikes!
DPatrick
2012-02-08 20:37:03
Computing these numbers seems really really unpleasant.
Computing these numbers seems really really unpleasant.
DPatrick
2012-02-08 20:37:27
In fact, so unpleasant that it's probably unreasonable to expect that computing these numbers is part of the solution.
In fact, so unpleasant that it's probably unreasonable to expect that computing these numbers is part of the solution.
DPatrick
2012-02-08 20:37:50
So instead of computing, let's try to make some qualitative observations first. (Meaning, what can we observe without bogging down into computations?)
So instead of computing, let's try to make some qualitative observations first. (Meaning, what can we observe without bogging down into computations?)
DPatrick
2012-02-08 20:38:07
Let's start simple: what do we know about the a's?
Let's start simple: what do we know about the a's?
rdj5933mile5
2012-02-08 20:38:24
always less than 1
always less than 1
brokenfixer
2012-02-08 20:38:24
positive, less than 1
positive, less than 1
DPatrick
2012-02-08 20:38:34
First, we can observe that all the a's are positive numbers between 0 and 1. (This is easy to prove by induction: at each step we're taking a number between 0 and 1 and raising it to a power between 0 and 1.)
First, we can observe that all the a's are positive numbers between 0 and 1. (This is easy to prove by induction: at each step we're taking a number between 0 and 1 and raising it to a power between 0 and 1.)
DPatrick
2012-02-08 20:39:11
What do we know about a_2? I don't want to compute it exactly of course, but in a vague sense what does it look like?
What do we know about a_2? I don't want to compute it exactly of course, but in a vague sense what does it look like?
brokenfixer
2012-02-08 20:39:40
(1/5)^(1/5)
(1/5)^(1/5)
hyperbolictangent
2012-02-08 20:39:42
almost 1
almost 1
Iggy Iguana
2012-02-08 20:39:44
.2^.2 is about .7 or .8?
.2^.2 is about .7 or .8?
DPatrick
2012-02-08 20:39:49
It's about 0.2^0.2. This is roughly the fifth root of 0.2. We expect this number to be close to 1, definitely well over 1/2.
It's about 0.2^0.2. This is roughly the fifth root of 0.2. We expect this number to be close to 1, definitely well over 1/2.
DPatrick
2012-02-08 20:40:05
So what does this tell us about a_3?
So what does this tell us about a_3?
number.sense
2012-02-08 20:40:38
a_3 is closer to 0.2
a_3 is closer to 0.2
mentalgenius
2012-02-08 20:40:38
closer to 0.2
closer to 0.2
DPatrick
2012-02-08 20:40:45
a_3 is basically 0.2 raised to a number close to 1. Therefore it's still pretty close to 0.2.
a_3 is basically 0.2 raised to a number close to 1. Therefore it's still pretty close to 0.2.
DPatrick
2012-02-08 20:41:07
Here's a key question, since we're concerned about ordering the numbers (to create the b's later): is a_3 smaller or bigger than a_1, and why?
Here's a key question, since we're concerned about ordering the numbers (to create the b's later): is a_3 smaller or bigger than a_1, and why?
strategist
2012-02-08 20:41:46
bigger because a_3 is bigger and a_2 is pretty much 1
bigger because a_3 is bigger and a_2 is pretty much 1
superpi83
2012-02-08 20:41:46
bigger because its exponent is less than 1.
bigger because its exponent is less than 1.
brokenfixer
2012-02-08 20:41:46
a_3 has bigger base and smaller power = bigger
a_3 has bigger base and smaller power = bigger
DPatrick
2012-02-08 20:42:18
Exactly, and that last comment by brokenfixer is the key: a_3 has a bigger base than a_1 and is raised to a smaller power than a_1 (think of a_1 as raised to the 1 power).
Exactly, and that last comment by brokenfixer is the key: a_3 has a bigger base than a_1 and is raised to a smaller power than a_1 (think of a_1 as raised to the 1 power).
DPatrick
2012-02-08 20:42:49
Both of those facts mean that a_3 is larger than a_1. (Since these numbers are between 0 and 1, the smaller the power, the larger the number.)
Both of those facts mean that a_3 is larger than a_1. (Since these numbers are between 0 and 1, the smaller the power, the larger the number.)
DPatrick
2012-02-08 20:43:01
What does this tell us about a_4? Can we use a similar argument?
What does this tell us about a_4? Can we use a similar argument?
DPatrick
2012-02-08 20:43:34
Is a_4 bigger than a_2 or smaller than a_2?
Is a_4 bigger than a_2 or smaller than a_2?
superpi83
2012-02-08 20:44:13
less than a_2 because it has a smaller base and a bigger power.
less than a_2 because it has a smaller base and a bigger power.
mathlover3737
2012-02-08 20:44:13
it's a larger exponent, so it's smaller
it's a larger exponent, so it's smaller
ClassClown
2012-02-08 20:44:13
small base, big power smaller overall
small base, big power smaller overall
DPatrick
2012-02-08 20:44:47
Right: a_4 is smaller than a_2 because it is being raised to a bigger power (namely, a_3) than a_2 was raised to (namely, a_1).
Right: a_4 is smaller than a_2 because it is being raised to a bigger power (namely, a_3) than a_2 was raised to (namely, a_1).
DPatrick
2012-02-08 20:45:02
What might we conjecture at this point?
What might we conjecture at this point?
saprmarks
2012-02-08 20:45:13
so are the a_(odds) getting larger as the a_(evens) getting smaller?
so are the a_(odds) getting larger as the a_(evens) getting smaller?
ClassClown
2012-02-08 20:45:17
it alternates smaller, larger, smaller, larger
it alternates smaller, larger, smaller, larger
DPatrick
2012-02-08 20:45:27
Exactly.
Exactly.
DPatrick
2012-02-08 20:45:37
The odd-position a's are increasing and the even-position a's are decreasing, and all the evens are bigger than all the odds.
The odd-position a's are increasing and the even-position a's are decreasing, and all the evens are bigger than all the odds.
DPatrick
2012-02-08 20:46:19
Can we prove this?
Can we prove this?
DPatrick
2012-02-08 20:46:35
Well, we can. But it's gory. I'm going to leave it as a homework exercise. :)
Well, we can. But it's gory. I'm going to leave it as a homework exercise. :)
DPatrick
2012-02-08 20:47:03
When actually taking the AMC's, I'd probably just convince myself that it's true but not rigorously prove it.
When actually taking the AMC's, I'd probably just convince myself that it's true but not rigorously prove it.
DPatrick
2012-02-08 20:47:41
So now we can go and try to finish the problem.
So now we can go and try to finish the problem.
DPatrick
2012-02-08 20:48:15
Let's use what we've learned, and list the original sequence and the new ordered sequence:
Let's use what we've learned, and list the original sequence and the new ordered sequence:
DPatrick
2012-02-08 20:48:43
Read the question carefully: the b's are in decreasing order. So a_2 (the biggest) comes first and a_1 (the smallest) comes last.
Read the question carefully: the b's are in decreasing order. So a_2 (the biggest) comes first and a_1 (the smallest) comes last.
DPatrick
2012-02-08 20:48:52
What goes in the question marks in the middle?
What goes in the question marks in the middle?
strategist
2012-02-08 20:49:11
1005 1006 1007 lol
1005 1006 1007 lol
rdj5933mile5
2012-02-08 20:49:11
a_1005,a_1006,a_1007
a_1005,a_1006,a_1007
osmosis92
2012-02-08 20:49:11
1005,1006,1007
1005,1006,1007
DPatrick
2012-02-08 20:49:16
There are 1005 even-indexed terms, so the first "?" is 1005:
There are 1005 even-indexed terms, so the first "?" is 1005:
strategist
2012-02-08 20:49:28
there will only be an intersection when b_k is coming down
there will only be an intersection when b_k is coming down
DPatrick
2012-02-08 20:49:46
Aha, we can see that none of the terms match up in the first 1005, so the other match(es) are in the later terms.
Aha, we can see that none of the terms match up in the first 1005, so the other match(es) are in the later terms.
Iggy Iguana
2012-02-08 20:50:01
and there will only be one such intersection
and there will only be one such intersection
delta1
2012-02-08 20:50:03
only one match
only one match
DPatrick
2012-02-08 20:50:07
Right.
Right.
DPatrick
2012-02-08 20:50:39
We could use a little test-taking strategy and see that the only plausible answer choice is the choice in the "middle" of the range from 1005 to 2011, so it must be (C) 1341.
We could use a little test-taking strategy and see that the only plausible answer choice is the choice in the "middle" of the range from 1005 to 2011, so it must be (C) 1341.
DPatrick
2012-02-08 20:50:55
But it's easy to verify it (and it's a good check that we didn't make a mistake).
But it's easy to verify it (and it's a good check that we didn't make a mistake).
DPatrick
2012-02-08 20:51:42
And it is (C) just like we suspected.
And it is (C) just like we suspected.
DPatrick
2012-02-08 20:52:08
Lastly, AMC 12A #25.
Lastly, AMC 12A #25.
DPatrick
2012-02-08 20:52:34
How can we approach this problem?
How can we approach this problem?
dlennon
2012-02-08 20:52:58
Graph f(x)?
Graph f(x)?
DPatrick
2012-02-08 20:53:15
That's a good idea. We want to try to understand what this function f(x) is doing, and graphing it is one way to do that.
That's a good idea. We want to try to understand what this function f(x) is doing, and graphing it is one way to do that.
narto928
2012-02-08 20:54:14
it's a V shape
it's a V shape
brokenfixer
2012-02-08 20:54:14
V shape
V shape
mentalgenius
2012-02-08 20:54:14
it is a V-shape
it is a V-shape
number.sense
2012-02-08 20:54:23
its an absolute value function that hits the x axis at 1/2
its an absolute value function that hits the x axis at 1/2
ClassClown
2012-02-08 20:54:23
It goes down until 1/2, then back p
It goes down until 1/2, then back p
Iggy Iguana
2012-02-08 20:54:58
it is repeating every increment of 1
it is repeating every increment of 1
DPatrick
2012-02-08 20:55:04
Right, so the graph looks like this:
Right, so the graph looks like this:
DPatrick
2012-02-08 20:55:41
We also notice that the range of f is just [0,1]. It's 0 at the points in H and it's 1 at the integers.
We also notice that the range of f is just [0,1]. It's 0 at the points in H and it's 1 at the integers.
brokenfixer
2012-02-08 20:55:50
what about xf(x)
what about xf(x)
number.sense
2012-02-08 20:55:50
now whats x times f(x)
now whats x times f(x)
DPatrick
2012-02-08 20:55:59
Yep, that's next. What does the graph of xf(x) look like?
Yep, that's next. What does the graph of xf(x) look like?
number.sense
2012-02-08 20:56:24
its scaled
its scaled
strategist
2012-02-08 20:56:25
like a staircase with valleys in it
like a staircase with valleys in it
brokenfixer
2012-02-08 20:56:25
looks like x at integers, 0 at half ints
looks like x at integers, 0 at half ints
narto928
2012-02-08 20:56:25
vertically stretched*
vertically stretched*
DPatrick
2012-02-08 20:56:55
Right. We're multiplying by x, so the "peaks", instead of being at 1, are now at x (for any positive integer x). It's very hard to graph it exactly but we can sketch it roughly:
Right. We're multiplying by x, so the "peaks", instead of being at 1, are now at x (for any positive integer x). It's very hard to graph it exactly but we can sketch it roughly:
DPatrick
2012-02-08 20:57:32
I drew them dashed because they're definitely not straight lines (in fact the pieces are parabolas), but they rise up from the x-axis at a point in H, to the next peak at an integer, and then back down to the x-axis at the next point in H. The only points we really know for sure are the points on the x-axis at the half-integers (where x is in H and y=0), and the points (x,x) where x is an integer.
I drew them dashed because they're definitely not straight lines (in fact the pieces are parabolas), but they rise up from the x-axis at a point in H, to the next peak at an integer, and then back down to the x-axis at the next point in H. The only points we really know for sure are the points on the x-axis at the half-integers (where x is in H and y=0), and the points (x,x) where x is an integer.
ClassClown
2012-02-08 20:57:52
how come the first one isn't a triangle
how come the first one isn't a triangle
DPatrick
2012-02-08 20:58:24
At x=0 we're multiply by 0, so we get the point (0,0) on the graph of xf(x). Then we get another zero at x=1/2 since f(1/2) = 0, and from there it proceeds "normally" (if you can call this normal).
At x=0 we're multiply by 0, so we get the point (0,0) on the graph of xf(x). Then we get another zero at x=1/2 since f(1/2) = 0, and from there it proceeds "normally" (if you can call this normal).
DPatrick
2012-02-08 20:58:54
I don't know how high the "peak" is (compared with the other peaks, which we do know accurately), so I drew it curved instead.
I don't know how high the "peak" is (compared with the other peaks, which we do know accurately), so I drew it curved instead.
DPatrick
2012-02-08 20:59:16
Now what happens when we apply x again? What's the graph of f(xf(x)) look like?
Now what happens when we apply x again? What's the graph of f(xf(x)) look like?
number.sense
2012-02-08 20:59:31
you mean apply f again?
you mean apply f again?
DPatrick
2012-02-08 20:59:37
Yeah, sorry, we apply **f** again.
Yeah, sorry, we apply **f** again.
DPatrick
2012-02-08 21:00:05
Again, remember that the integers evaluate to 1 when we apply f. So the point x=0 and all the points in H, since they are zero in xf(x), will evaluate to 1 in f(xf(x)).
Again, remember that the integers evaluate to 1 when we apply f. So the point x=0 and all the points in H, since they are zero in xf(x), will evaluate to 1 in f(xf(x)).
number.sense
2012-02-08 21:00:15
wherever the function was a half integer, it goes down to 0, wheever it is 0, it goes up to 1
wherever the function was a half integer, it goes down to 0, wheever it is 0, it goes up to 1
DPatrick
2012-02-08 21:00:38
Right, but moreover, wherever the function xf(x) was any integer (not just 0), the function f(xf(x)) will be 1.
Right, but moreover, wherever the function xf(x) was any integer (not just 0), the function f(xf(x)) will be 1.
DPatrick
2012-02-08 21:00:45
It might be clearer if I put some red and blue bands in my previous graph:
It might be clearer if I put some red and blue bands in my previous graph:
DPatrick
2012-02-08 21:01:11
The points on the red bands (with y-values in the integers) will go to 1 when I apply f again.
The points on the red bands (with y-values in the integers) will go to 1 when I apply f again.
brokenfixer
2012-02-08 21:01:14
red 1 blue 0
red 1 blue 0
DPatrick
2012-02-08 21:01:17
Yep.
Yep.
DPatrick
2012-02-08 21:01:29
The points on the blue bands (with y-values in H, the half-integers) will go to 0 when I apply f again.
The points on the blue bands (with y-values in H, the half-integers) will go to 0 when I apply f again.
DPatrick
2012-02-08 21:01:49
So we just count where we cross the red and blue bands.
So we just count where we cross the red and blue bands.
DPatrick
2012-02-08 21:01:57
Again, the red points go to 1, and the blue points go to 0.
Again, the red points go to 1, and the blue points go to 0.
DPatrick
2012-02-08 21:02:07
We get one cycle from 1 to 0 to 1 in the interval [0,1]
We get three cycles from 1 to 0 to 1 to 0 to 1 to 0 to 1 in the interval [1,2]
We get five cycles from 1 to 0 to 1 to 0 to 1 to 0 to 1 to 0 to 1 to 0 to 1 in the interval [2,3]
And so on.
We get one cycle from 1 to 0 to 1 in the interval [0,1]
We get three cycles from 1 to 0 to 1 to 0 to 1 to 0 to 1 in the interval [1,2]
We get five cycles from 1 to 0 to 1 to 0 to 1 to 0 to 1 to 0 to 1 to 0 to 1 in the interval [2,3]
And so on.
DPatrick
2012-02-08 21:02:23
...and it looks something like this (I'm going to widen it and only show up to x=3 to hopefully make it look a little clearer):
...and it looks something like this (I'm going to widen it and only show up to x=3 to hopefully make it look a little clearer):
DPatrick
2012-02-08 21:02:37
Again, they're not straight lines, and they're not this "regular".
Again, they're not straight lines, and they're not this "regular".
DPatrick
2012-02-08 21:03:04
I'm only trying to capture the broad behavior: where it's 0 and where it's 1.
I'm only trying to capture the broad behavior: where it's 0 and where it's 1.
DPatrick
2012-02-08 21:03:20
This is only a qualitative picture of the function f(xf(x)), and that's good enough for the next step.
This is only a qualitative picture of the function f(xf(x)), and that's good enough for the next step.
brokenfixer
2012-02-08 21:03:30
compare to y = x/n?
compare to y = x/n?
number.sense
2012-02-08 21:03:33
now we want this to intersect the line y = x/n 2012 times
now we want this to intersect the line y = x/n 2012 times
DPatrick
2012-02-08 21:03:47
Exactly -- we now figure out when this equals x/n for some n.
Exactly -- we now figure out when this equals x/n for some n.
DPatrick
2012-02-08 21:04:06
Since n is pretty big, x/n is a line through (0,0) with pretty low slope. I'll add it to the previous picture:
Since n is pretty big, x/n is a line through (0,0) with pretty low slope. I'll add it to the previous picture:
DPatrick
2012-02-08 21:04:19
What do you notice?
What do you notice?
KeepingItReal
2012-02-08 21:04:50
intersects 2 times per triangle
intersects 2 times per triangle
Iggy Iguana
2012-02-08 21:04:50
intersections are 2, 6, 10, 14, ...
intersections are 2, 6, 10, 14, ...
brokenfixer
2012-02-08 21:04:50
2,6,10,14,...
2,6,10,14,...
314159Nerd
2012-02-08 21:04:50
It intersects 2 * x^2 times by the time it reaches an x value
It intersects 2 * x^2 times by the time it reaches an x value
DPatrick
2012-02-08 21:04:58
There are 2 solutions on the interval [0,1].
There are 6 solutions on the interval [1,2].
There are 10 solutions on the interval [2,3].
There are 2 solutions on the interval [0,1].
There are 6 solutions on the interval [1,2].
There are 10 solutions on the interval [2,3].
DPatrick
2012-02-08 21:05:38
...since we have the sum 2(1+3+5+...+(2k-1)), which is 2k^2.
...since we have the sum 2(1+3+5+...+(2k-1)), which is 2k^2.
DPatrick
2012-02-08 21:06:10
Of course, we also know that when nx > 1, there can't be any more solutions.
Of course, we also know that when nx > 1, there can't be any more solutions.
narto928
2012-02-08 21:06:20
Set 2k^2 = 2012 and solve, round up for "at least"
Set 2k^2 = 2012 and solve, round up for "at least"
alex31415
2012-02-08 21:06:20
find k such that 2k^2>=2012?
find k such that 2k^2>=2012?
DPatrick
2012-02-08 21:06:48
Those are weird symbols.... :(
Those are weird symbols.... :(
DPatrick
2012-02-08 21:07:07
Just to be sure, we should check around our answer.
Just to be sure, we should check around our answer.
DPatrick
2012-02-08 21:07:12
If we use n=31, we'll only get 2(31)^2 = 1922 solutions, not enough.
If we use n=31, we'll only get 2(31)^2 = 1922 solutions, not enough.
DPatrick
2012-02-08 21:07:19
If we use n=32, we'll get 2(32)^2 = 2048 solutions, more than enough!
If we use n=32, we'll get 2(32)^2 = 2048 solutions, more than enough!
number.sense
2012-02-08 21:07:39
which gives us 32
which gives us 32
alex31415
2012-02-08 21:07:39
the answer is 32?
the answer is 32?
DPatrick
2012-02-08 21:07:43
So n=32 is the smallest n giving us at least 2012 solutions. Answer (C).
So n=32 is the smallest n giving us at least 2012 solutions. Answer (C).
Iggy Iguana
2012-02-08 21:07:47
there are a lot of (c) answers
there are a lot of (c) answers
DPatrick
2012-02-08 21:07:57
Yes, the answer to each of 22 through 25 on the AMC 12A was (C).
Yes, the answer to each of 22 through 25 on the AMC 12A was (C).
DPatrick
2012-02-08 21:08:29
Notice the strategy -- we didn't exactly determine what f(xf(x)) looks liked, and we certainly didn't compute it. We just got an accurate-enough qualitative picture to allow us to count the roots.
Notice the strategy -- we didn't exactly determine what f(xf(x)) looks liked, and we certainly didn't compute it. We just got an accurate-enough qualitative picture to allow us to count the roots.
DPatrick
2012-02-08 21:08:58
We've run for 2 hours, but I will let you request **one** additional problem (say which test it is from), and I'll pick the one that gets the most vote and/or I like the most. :)
We've run for 2 hours, but I will let you request **one** additional problem (say which test it is from), and I'll pick the one that gets the most vote and/or I like the most. :)
DPatrick
2012-02-08 21:10:00
Hang on, I'm tabulating the votes...
Hang on, I'm tabulating the votes...
DPatrick
2012-02-08 21:10:40
Sadly, no one picked my favorite (which was #16 on the 12A), and the overwhelming winner was the "Paula the Painter" problem, so here goes:
Sadly, no one picked my favorite (which was #16 on the 12A), and the overwhelming winner was the "Paula the Painter" problem, so here goes:
DPatrick
2012-02-08 21:11:09
oops, wrong problem!
oops, wrong problem!
ss5188
2012-02-08 21:11:54
set up systems of equations
set up systems of equations
DPatrick
2012-02-08 21:12:13
The trick with this sort of "work" problem is setting up the right equations. From there the algebra is usually pretty easy.
The trick with this sort of "work" problem is setting up the right equations. From there the algebra is usually pretty easy.
Draco
2012-02-08 21:12:17
We can have the rates of the helpers be 1 variable.
We can have the rates of the helpers be 1 variable.
DPatrick
2012-02-08 21:12:25
That simplifies things a little bit.
That simplifies things a little bit.
soccerfan
2012-02-08 21:12:29
3 equations, 3 variables
3 equations, 3 variables
DPatrick
2012-02-08 21:12:36
Say that Paula paints p% of the house in 1 minute, and the helpers (together) paint h% of the house in 1 minute. Also say the lunch break is m minutes.
Say that Paula paints p% of the house in 1 minute, and the helpers (together) paint h% of the house in 1 minute. Also say the lunch break is m minutes.
DPatrick
2012-02-08 21:12:51
We might as well use percentages for p and h since the given data in the problem is also in percentages.
We might as well use percentages for p and h since the given data in the problem is also in percentages.
ahaanomegas
2012-02-08 21:13:05
(p+h)(480-m) = 50
(p+h)(480-m) = 50
DPatrick
2012-02-08 21:13:53
Those are the three equations, and what's left is to solve for m.
Those are the three equations, and what's left is to solve for m.
strategist
2012-02-08 21:14:31
after a bit of tedious algebra, it is trivialized
after a bit of tedious algebra, it is trivialized
DPatrick
2012-02-08 21:14:55
That's kind of my opinion. I didn't actually work out the algebra from this point.
That's kind of my opinion. I didn't actually work out the algebra from this point.
DPatrick
2012-02-08 21:15:09
I think that most of the difficulty is the setup.
I think that most of the difficulty is the setup.
DPatrick
2012-02-08 21:15:55
We're now a bit over 2 hours, so I think I'll stop here with some closing remarks.
We're now a bit over 2 hours, so I think I'll stop here with some closing remarks.
DPatrick
2012-02-08 21:16:13
First, I have an exciting (I hope) announcement!
First, I have an exciting (I hope) announcement!
DPatrick
2012-02-08 21:16:21
Our own Richard Rusczyk has been making video solutions for many of this year's AMC 10A and 12A problems. These will be up on our website in the next few days. Click on the "Videos" tab of the website to see them when they're up (and of course we'll make an announcement on the Community when they are available). They will probably be on YouTube too.
Our own Richard Rusczyk has been making video solutions for many of this year's AMC 10A and 12A problems. These will be up on our website in the next few days. Click on the "Videos" tab of the website to see them when they're up (and of course we'll make an announcement on the Community when they are available). They will probably be on YouTube too.
DPatrick
2012-02-08 21:17:01
Second, as you probably know, the AMC 10B/12B is on Wednesday, February 22 (exactly two weeks from today). As we did today, we'll have our Math Jam the day after, Feb 23, at 7 PM Eastern.
Second, as you probably know, the AMC 10B/12B is on Wednesday, February 22 (exactly two weeks from today). As we did today, we'll have our Math Jam the day after, Feb 23, at 7 PM Eastern.
DPatrick
2012-02-08 21:17:18
I believe that my colleague Jeremy Copeland will be leading the discussion on the 23rd.
I believe that my colleague Jeremy Copeland will be leading the discussion on the 23rd.
DPatrick
2012-02-08 21:17:47
Next, a shameless plug for AoPS. :)
Next, a shameless plug for AoPS. :)
DPatrick
2012-02-08 21:17:51
If you qualify for the AIME, congratulations! AoPS will be holding our annual Special AIME Problem Seminar on the weekend of March 3 and 4, from 3:30 to 6:30 (Eastern Time) both days, taught by Richard Rusczyk and myself. Each day consists of general approaches and important facts needed for problems within a given subject area, followed by a discussion of specific problems from past AIME competitions, or from other contests of a similar difficulty level. This course is largely a repeat of the weekend seminar we offered in 2010, and contains some material from the AIME Problem Series A course. This course is appropriate for students who are hoping to pass the AIME and qualify for the USA(J)MO. If a student already consistently scores above 10 on the AIME, this class is probably not necessary, and if a student is unlikely to answer more than 1 or 2 questions correctly, then that student should start with some of our Introduction series of classes. The cost is $75. You can enroll by clicking on the "School" tab of the website, and selecting "Special AIME Problem Seminar" from the list of courses.l
If you qualify for the AIME, congratulations! AoPS will be holding our annual Special AIME Problem Seminar on the weekend of March 3 and 4, from 3:30 to 6:30 (Eastern Time) both days, taught by Richard Rusczyk and myself. Each day consists of general approaches and important facts needed for problems within a given subject area, followed by a discussion of specific problems from past AIME competitions, or from other contests of a similar difficulty level. This course is largely a repeat of the weekend seminar we offered in 2010, and contains some material from the AIME Problem Series A course. This course is appropriate for students who are hoping to pass the AIME and qualify for the USA(J)MO. If a student already consistently scores above 10 on the AIME, this class is probably not necessary, and if a student is unlikely to answer more than 1 or 2 questions correctly, then that student should start with some of our Introduction series of classes. The cost is $75. You can enroll by clicking on the "School" tab of the website, and selecting "Special AIME Problem Seminar" from the list of courses.l
DPatrick
2012-02-08 21:18:26
We will also have Math Jams for each of the two AIME contests in March -- check the website for specific dates and times.
We will also have Math Jams for each of the two AIME contests in March -- check the website for specific dates and times.
DPatrick
2012-02-08 21:19:01
I will stick around for a little while to answer questions, but please: I will not speculate on the cutoff score for AIME qualification, so please don't ask me to.
I will stick around for a little while to answer questions, but please: I will not speculate on the cutoff score for AIME qualification, so please don't ask me to.
DPatrick
2012-02-08 21:19:26
And a reminder: the transcript of our entire session will be posted on the website as soon as I wrap up and finish answering questions.
And a reminder: the transcript of our entire session will be posted on the website as soon as I wrap up and finish answering questions.
DPatrick
2012-02-08 21:19:35
That's all for tonight -- thanks for coming!
That's all for tonight -- thanks for coming!
hrithikguy
2012-02-08 21:19:45
will this year's AIME seminar be different from last year's?
will this year's AIME seminar be different from last year's?
DPatrick
2012-02-08 21:20:11
Yes, it is completely different material than 2011 -- it cycles every 2 years (so if you took it in 2010 it's largely a repeat).
Yes, it is completely different material than 2011 -- it cycles every 2 years (so if you took it in 2010 it's largely a repeat).
Klu2014
2012-02-08 21:20:42
So they are based on the same topics?
So they are based on the same topics?
DPatrick
2012-02-08 21:21:05
It's the same four general topics (algebra, counting, geometry, and number theory), but the problems used to illustrate the material and techniques are all different in 2012 than from 2011.
It's the same four general topics (algebra, counting, geometry, and number theory), but the problems used to illustrate the material and techniques are all different in 2012 than from 2011.
dsj9999
2012-02-08 21:21:15
Do you know when we'll learn the cutoff though?
Do you know when we'll learn the cutoff though?
vliu
2012-02-08 21:21:15
when should the official scores for our AMC's come out?
when should the official scores for our AMC's come out?
DPatrick
2012-02-08 21:21:21
I don't know.
I don't know.
KeepingItReal
2012-02-08 21:21:27
Do you think this year's contest was harder than normal, or was it trivial as strategist said?
Do you think this year's contest was harder than normal, or was it trivial as strategist said?
BOGTRO
2012-02-08 21:21:27
What do you think the relative difficulty of this year was?
What do you think the relative difficulty of this year was?
strategist
2012-02-08 21:21:27
how hard was this compared to last years amc?
how hard was this compared to last years amc?
DPatrick
2012-02-08 21:21:51
This is a topic I'd prefer not to comment on, because "harder" for some people is "easier" for others. Also I haven't read all the problems. :)
This is a topic I'd prefer not to comment on, because "harder" for some people is "easier" for others. Also I haven't read all the problems. :)
Klu2014
2012-02-08 21:22:00
Do you write the AMC tests?
Do you write the AMC tests?
DPatrick
2012-02-08 21:22:19
No. I also didn't see the test until yesterday, just like you.
No. I also didn't see the test until yesterday, just like you.
Flamewire
2012-02-08 21:22:28
What's the best way to prepare for the AMC12, by doing old tests? What types of topics should we learn?
What's the best way to prepare for the AMC12, by doing old tests? What types of topics should we learn?
DPatrick
2012-02-08 21:22:47
Taking old tests is a great way to practice, both for the material and for "test-taking" skills like pacing yourself, checking your work, and so on.
Taking old tests is a great way to practice, both for the material and for "test-taking" skills like pacing yourself, checking your work, and so on.
Burtleberry
2012-02-08 21:23:10
Do you think the AOPS Volume II is good preparation for AIME? Any other reading recommendations?
Do you think the AOPS Volume II is good preparation for AIME? Any other reading recommendations?
hrithikguy
2012-02-08 21:23:10
how do you get faster at doing computation on AMC problems?
how do you get faster at doing computation on AMC problems?
distortedwalrus
2012-02-08 21:23:10
what should we do when we run out of old tests to take?
what should we do when we run out of old tests to take?
DPatrick
2012-02-08 21:23:40
You should ask these questions on the AMC Forum on our website. Other students will have better recommendations than I.
You should ask these questions on the AMC Forum on our website. Other students will have better recommendations than I.
KeepingItReal
2012-02-08 21:24:12
If the AIME qualifying index for 12A is lower than normal, will the USAMO qualifying index be lower than normal?
If the AIME qualifying index for 12A is lower than normal, will the USAMO qualifying index be lower than normal?
DPatrick
2012-02-08 21:24:42
Administrative questions like this should be directed to the AMC themselves, or ask on the forum. I try not to get bogged down in AMC minutia.
Administrative questions like this should be directed to the AMC themselves, or ask on the forum. I try not to get bogged down in AMC minutia.
breakeroice
2012-02-08 21:24:58
If I am a sophomore in high school, should I take the AMC10B or 12B if given the option to take either?
If I am a sophomore in high school, should I take the AMC10B or 12B if given the option to take either?
DPatrick
2012-02-08 21:25:33
You should search for this on the forum (or re-ask it): lots of people have lots of different opinions. My only short answer is "it depends" (on what your realistic goals are).
You should search for this on the forum (or re-ask it): lots of people have lots of different opinions. My only short answer is "it depends" (on what your realistic goals are).
monarchb
2012-02-08 21:25:54
Do you get the official answers from MAA for the AMC10 or do you work them out?
Do you get the official answers from MAA for the AMC10 or do you work them out?
DPatrick
2012-02-08 21:26:23
We got the official answers, the same as your school did. But I didn't look at them until after I had solved the problem, as a check of my work and to see if they had done anything more clever than I did.
We got the official answers, the same as your school did. But I didn't look at them until after I had solved the problem, as a check of my work and to see if they had done anything more clever than I did.
DPatrick
2012-02-08 21:26:59
Fair warning: I am going to stop at :30 past, and you will be forcefully evicted from the classroom so I can generate the transcript. (Sorry.)
Fair warning: I am going to stop at :30 past, and you will be forcefully evicted from the classroom so I can generate the transcript. (Sorry.)
math-rules
2012-02-08 21:27:37
is this Q and A part recorded on the transcript of this math jam?
is this Q and A part recorded on the transcript of this math jam?
DPatrick
2012-02-08 21:27:39
It will be.
It will be.
KeepingItReal
2012-02-08 21:28:08
wait, my school has the official answers?
wait, my school has the official answers?
mentalgenius
2012-02-08 21:28:09
how can I get a hold of the solutions for this year's AMC 10 A? (They usually hand out the solutions at the actual contest, but they didn't at school this time for some reason)
how can I get a hold of the solutions for this year's AMC 10 A? (They usually hand out the solutions at the actual contest, but they didn't at school this time for some reason)
DPatrick
2012-02-08 21:28:22
Every school that administered the contest has a copy of the solutions.
Every school that administered the contest has a copy of the solutions.
DPatrick
2012-02-08 21:28:44
They cannot be posted online, as they are copyrighted by the AMC.
They cannot be posted online, as they are copyrighted by the AMC.
ahaanomegas
2012-02-08 21:28:55
It's called the "Solutions Pamphlet", in blue.
It's called the "Solutions Pamphlet", in blue.
Bictor717
2012-02-08 21:29:01
The rules say that solutions are not to be given out until the answer sheets are mailed.
The rules say that solutions are not to be given out until the answer sheets are mailed.
DPatrick
2012-02-08 21:29:10
Yes -- your school should have mailed them today.
Yes -- your school should have mailed them today.
breakeroice
2012-02-08 21:29:14
Is the reason why the Math Jam is today because you aren't allowed to talk about the test until the day after?
Is the reason why the Math Jam is today because you aren't allowed to talk about the test until the day after?
DPatrick
2012-02-08 21:29:16
Yes.
Yes.
distortedwalrus
2012-02-08 21:29:24
what did you get on the AMC when you took it?
what did you get on the AMC when you took it?
DPatrick
2012-02-08 21:29:30
My senior year, 150. :)
My senior year, 150. :)
DPatrick
2012-02-08 21:29:41
That seems like a great place to stop. Good night!
That seems like a great place to stop. Good night!
Copyright © 2024 AoPS Incorporated. This page is copyrighted material. You can view and print this page for your own use, but you cannot share the contents of this file with others.